Criminal Law

अब Quizwiz के साथ अपने होमवर्क और परीक्षाओं को एस करें!

In order to be guilty of common law burglary, one must break and enter a dwelling with: response - correct A Recklessness about the consequences that could occur if someone was inside the home Correct B The specific intent to commit a felony therein C Knowledge that one lacks authority to enter the dwelling D No particular mental state; the act of breaking and entering is sufficient to establish burglary

Burglary is a specific intent crime. A specific intent crime requires the doing of the criminal act with a specific intent or objective. The common law definition of burglary is the breaking and entering of the dwelling of another at nighttime with the intent of committing a felony therein. Because burglary requires the intent to commit a felony at the time of the entering, it is a specific intent crime.

A man and a woman were arrested and charged with conspiring to blow up a federal government building. After being given Miranda warnings, they were questioned separately and each of them gave a written confession. The confessions interlocked with each other, implicating both of the defendants as being involved in every stage of the conspiracy. Subsequently, the woman attempted to retract her confession, claiming that it was false. At a preliminary hearing, the judge rejected her claim. Both defendants were tried together, and the prosecutor introduced both confessions into evidence. At trial, the woman testified that she was not involved in any conspiracy and that her confession was fabricated. Both defendants were found guilty by the jury. The woman challenges her conviction on appeal because of the admission of the man's confession. If the woman succeeds, what is the likely reason? response - correct AThe man's confession was more incriminatory to her than her own confession. BThe jury was not instructed to consider the man's confession as evidence only of his guilt and not of the woman's. Correct CThe man refused to testify at trial and therefore was not subject to cross-examination regarding his confession. DThe man testified at trial and was subject to cross-examination but denied making the confession attributed to him.

If the woman prevails in her challenge to the admission of the man's confession, it will be because the man could not be cross-examined regarding his confession. Under the Sixth Amendment, a defendant in a criminal prosecution has the right to confront adverse witnesses at trial. If two persons are tried together and one has given a confession that implicates the other, the right of confrontation generally prohibits the use of that statement because the other defendant cannot compel the confessing co-defendant to take the stand for cross-examination. A co-defendant's confession is inadmissible even when it interlocks with the defendant's own confession, which is admitted. If the man refused to take the stand and subject himself to cross-examination, his confession was not properly admitted because it violated the woman's Confrontation Clause rights. (A) is incorrect because the fact that the man's confession incriminates the woman more than her own confession is not relevant. Just the interlocking nature of the man's confession with the woman's confession makes it more damaging by making it harder for the woman to claim that her confession was false. (B) is incorrect because the Supreme Court has held that instructing the jury to consider the confession only as going to the guilt of the confessing defendant is inadequate to avoid Confrontation Clause problems, because the risk that the jury will not follow the limiting instructions is too great in this context. (D) is incorrect. Confessions of a co-defendant may be admitted if (i) all portions referring to the other defendant can be eliminated (so that there is no indication of that defendant's involvement), (ii) the confessing defendant takes the stand and subjects himself to cross-examination regarding the truth or falsity of the statement, or (iii) the confession of the nontestifying co-defendant is being used to rebut the defendant's claim that his confession was obtained coercively, and the jury is instructed as to that purpose. Even if the co-defendant denies ever having made the confession, as stated in choice (D), the opportunity at trial to cross-examine the co-defendant satisfies the Confrontation Clause.

An art restorer, after attending art school for a number of years, secured a job restoring the paintings for an art museum. After several years on the job, the artist discovered that he could imitate the artwork of nearly any artist. He decided that he could make some extra money copying the artwork of up-and-coming artists, while staying away from more well-known artists to reduce his chance of getting caught. An art collector searching for a painting by a new artist saw the restorer at an art fair selling various paintings, one of which appeared to be by the new artist. The restorer was selling the painting for $100. The collector thought that the price was very low and that the painting should probably sell for around $500, but she bought the painting anyway, giving the restorer $100 after the restorer confirmed that the painting was an original from the new artist. After taking it to an art appraiser for insurance purposes, she discovered that the painting was a forgery. However, she also discovered that the painting's frame was worth about $125. With which theft offense may the art restorer be charged? response - correct ALarceny by trick. Correct BFalse pretenses. CEmbezzlement. DNo theft offense.

The art restorer has committed false pretenses because his misrepresentation concerning the authenticity of the painting induced the collector to convey title to the $100. The offense of false pretenses consists of obtaining title to the property of another by an intentional (or knowing) false statement of past or existing fact, with intent to defraud the other. The art restorer falsely represented to the collector that the painting he sold her was an original from a new artist, intending that the collector would rely on such a misrepresentation by paying money for the painting. The collector, acting in reliance on this misrepresentation, conveyed to the art restorer title to the $100. Thus, all of the elements of false pretenses are present in the restorer's dealings with the collector. Because the restorer has committed false pretenses, which is a theft offense, (D) is incorrect. Monetary loss on the part of the victim is not an element of the crime. Thus, although the lack of damages might prevent the collector from suing in civil court, the fact that the collector suffered no monetary loss from the crime is irrelevant for criminal law purposes. (A) is incorrect because the restorer obtained title to the $100 rather than mere possession. If a victim consents to someone's taking possession of property, but such consent is induced by a misrepresentation, the consent is not valid. The resulting offense is larceny by trick. False pretenses differs from larceny by trick in what is obtained. If the victim intends to convey only possession of the property, the offense is larceny by trick. However, if the victim intends to convey title, the offense is false pretenses. Here, the collector intended to convey title to the $100, acting in reliance on the restorer's false representation that the painting was an original. Because the restorer obtained title, the offense of which he can be convicted is false pretenses rather than larceny by trick. (C) is incorrect because embezzlement consists of the fraudulent conversion of property of another by a person in lawful possession of that property. In embezzlement, misappropriation of the property occurs while the defendant has lawful possession of it. Here, the restorer did not convert the $100 while he was in lawful possession of it; rather, he obtained title to the money by means of a misrepresentation. Because the restorer did not have lawful possession of the money, he has not committed embezzlement.

In most states, murder is of the second degree unless __________, in which case the murder is of the first degree. response - correct A deliberation and premeditation can be shown, or the defendant acted "with a depraved heart" Correct B deliberation and premeditation can be shown, or the killing occurred during an enumerated felony C the killing occurred during an enumerated felony, or the defendant acted "with a depraved heart" D "a depraved heart" can be shown

In most states, murder is of the second degree unless deliberation and premeditation can be shown, or the killing occurred during an enumerated felony, in which case the murder is of the first degree. If neither can be shown, the killing will usually be second degree murder (unless the killing is downgraded to manslaughter based on adequate provocation). Depraved heart murder would ordinarily be second degree murder in most states.

The following are elements of larceny: response - correct A Taking of personal property; passing of title; trespass Correct B Taking of personal property; asportation; trespass C Taking of personal property; asportation; passing of title D Asportation; passing of title; trespass

Larceny consists of: (i) A taking; (ii) And carrying away (asportation); (iii) Of tangible personal property; (iv) Of another; (v) By trespass; (vi) With intent to permanently (or for an unreasonable time) deprive the person of his interest in the property. The passing of title is not an element of larceny. However, it is an element of false pretenses.

Miranda warnings __________ need to be given before a suspect is interrogated by a civilian working for the police. response - correct A Always B Never Correct C May

Miranda warnings may need to be given, depending on whether the suspect knows this person is employed by the police. Miranda generally applies only to interrogation by the publicly paid police. It does not apply where interrogation is by an informant who the defendant does not know is working for the police. The rationale is that the warnings are intended to offset the coercive nature of police-dominated interrogation, and if the defendant does not know that he is being interrogated by the police, there is no coercive atmosphere to offset.

A defendant was convicted after a jury trial of violation of federal statutes prohibiting the sale of automatic weapons to foreign nationals. It was established at trial that the defendant had purchased a number of stolen United States Army heavy machine guns and attempted to ship them abroad. The trial court expressly based its imposition of the maximum possible sentence for the conviction on the defendant's refusal to reveal the names of the persons from whom he purchased the stolen weapons. His counsel argues that this consideration is reversible error. If the defendant appeals the sentence imposed, what should the appeals court do? response - incorrect Incorrect AReverse the trial court, because the consideration of the defendant's silence violates his Fifth Amendment privilege against self-incrimination. BReverse the trial court, because the consideration of collateral circumstances in sentencing violates his due process rights. Correct CAffirm the trial court, because the right to remain silent granted by the Fifth Amendment does not include the right to protect others from incrimination. DAffirm the trial court, because citizens must report violations of the criminal statutes.

The appeals court should affirm the trial court because the right to remain silent does not include the right to protect others from incrimination. The defendant was not privileged to refuse revealing the names of the stolen weapon sellers. The United States Supreme Court held, in Roberts v. United States (1980), that a defendant's refusal to cooperate with an investigation of the criminal conspiracy of which he was a member may properly be considered in imposing sentence. This is because the Fifth Amendment right to remain silent does not afford a privilege to refuse to incriminate others. (C) is therefore correct and (A) is incorrect. (B) is incorrect because the court's consideration of the defendant's refusal to cooperate does not violate due process. (D) is not an accurate statement of the law.

With regard to the crime of robbery, which of the following statements is true? response - correct A The property must be taken from the victim's person. B If threats of immediate death or serious physical injury are used, they must be threats only to the robbery victim. Correct C The force or threats of force may be used to retain possession immediately after such possession has been accomplished. D If intimidation is used, a threat to destroy the victim's dwelling house is insufficient.

The force or threats may be used either to gain possession of the property or to retain possession immediately after such possession has been accomplished. The property must be taken from the victim's person or presence. "Presence" means some location reasonably close to the victim, but it need not be taken from the victim's person. Property in other rooms of the house in which the victim is located is in her "presence." If force is used, it must be sufficient to overcome the victim's resistance. If threats of immediate death or serious physical injury are used, they must be threats to the victim, a member of her family, a relative, or a person in her presence at the time. A threat to do damage to property will not suffice-with the exception of a threat to destroy the victim's dwelling house.

Voluntary manslaughter is a killing committed: response - correct A During the course of a crime not amounting to a felony B During the course of a felony C Without express malice Correct D Under adequate provocation

A killing committed under adequate provocation is voluntary manslaughter. A killing committed during the commission of an unlawful act not amounting to a felony is involuntary manslaughter. Committing a felony that results in death is murder. The intent to commit a felony satisfies the malice requirement to classify a killing as murder. A killing committed without express malice may be murder if malice is implied. "Malice aforethought" for common law murder can be satisfied by (i) the intent to kill; (ii) the intent to inflict great bodily injury; (iii) a reckless indifference to an unjustifiably high risk to human life; or (iv) the intent to commit a felony. The intent to kill is deemed to be "express malice," whereas, in the latter three, malice is implied. Although "implied," the latter three states of mind also satisfy the malice requirement for common law murder.

A conspirator can be convicted of a crime committed by another conspirator if: response - correct A The crimes were committed in furtherance of the objectives of the conspiracy. Correct B The crimes were committed in furtherance of the objectives of the conspiracy, and the crimes were foreseeable. C The crimes were committed in furtherance of the objectives of the conspiracy, or the crimes were foreseeable. D The crimes were foreseeable.

A conspirator can be convicted of a crime committed by another conspirator if the crimes were committed in furtherance of the objectives of the conspiracy AND the crimes were foreseeable.

For the crime of conspiracy, which statement best describes the necessary intent? response - incorrect AOnly the intent to achieve the objective of the conspiracy BThe intent to agree or the intent to achieve the objective of the conspiracy Incorrect COnly the intent to agree Correct DThe intent to agree and the intent to achieve the objective of the conspiracy

A defendant must possess both the intent to agree AND the intent to achieve the objective of the conspiracy to be convicted of conspiracy.

The fact that the defendant committed a particular act is sufficient for the jury to infer that he acted with: response - correct A Malice Correct B General intent C Specific intent D Negligence

A jury can infer the required general intent merely from the doing of the act. It is not necessary that evidence specifically proving the general intent be offered by the prosecution. However, a jury cannot infer that the defendant acted with specific intent by the doing of the act. (That said, the manner in which an act is done may provide circumstantial evidence that the defendant acted with specific intent.) Malice is established by showing that the defendant recklessly disregarded an obvious or high risk that a particular harmful result would occur. Negligence is established by showing that the defendant failed to be aware of a substantial and unjustifiable risk that circumstances existed or a result would follow, and such failure constituted a substantial deviation from the standard of care that a reasonable person would exercise under the circumstances.

The Model Penal Code defines acting purposefully as when: response - correct A The defendant is aware that his conduct is of that nature or that certain circumstances exist Correct B It is the defendant's conscious object to engage in certain conduct or cause a certain result C The defendant consciously disregards a substantial and unjustifiable risk that circumstances exist or that a prohibited result will follow, and this disregard constitutes a gross deviation from the standard of care that a reasonable person would exercise in the situation D The defendant fails to be aware of a substantial and unjustifiable risk that circumstances exist or a result will follow, and such failure constitutes a substantial deviation from the standard of care that a reasonable person would exercise under the circumstances

A person acts purposefully when it is his conscious object to engage in certain conduct or cause a certain result. A person acts negligently when he fails to be aware of a substantial and unjustifiable risk that circumstances exist or a result will follow, and such failure constitutes a substantial deviation from the standard of care that a reasonable person would exercise under the circumstances. A person acts knowingly when he is aware that his conduct is of that nature or that certain circumstances exist. A person acts recklessly when he consciously disregards a substantial and unjustifiable risk that circumstances exist or that a prohibited result will follow, and this disregard constitutes a gross deviation from the standard of care that a reasonable person would exercise in the situation. Recklessness is one of the four categories into which the Model Penal Code classifies the mental component of a criminal offense (i.e., the element of fault).

Under the Model Penal Code, consciously disregarding a substantial and unjustifiable risk indicates a person acted __________. response - correct Aknowingly Bpurposefully Correct Crecklessly Dnegligently

A person acts recklessly when he consciously disregards a substantial and unjustifiable risk that circumstances exist or that a prohibited result will follow, and this disregard constitutes a gross deviation from the standard of care that a reasonable person would exercise in the situation. Recklessness is one of the four categories into which the Model Penal Code classifies the mental component of a criminal offense (i.e., the element of fault). A person acts purposefully when it is his conscious object to engage in certain conduct or cause a certain result. A person acts knowingly when he is aware that his conduct is of that nature or that certain circumstances exist. A person acts negligently when he fails to be aware of a substantial and unjustifiable risk that circumstances exist or a result will follow, and such failure constitutes a substantial deviation from the standard of care that a reasonable person would exercise under the circumstances.

If more than _______ months' imprisonment is authorized, the offense is considered "serious" for determining whether a defendant has a constitutional right to a jury trial. response - correct A nine Correct B six C three D twelve

An offense is considered serious, making a jury trial a constitutional right, when more than six months' imprisonment is authorized.

The difference between the crimes of larceny by false pretenses and larceny "by trick" is that: response - correct A For larceny by false pretenses, the victim must actually rely on the misrepresentation in giving up her property, but for larceny by trick, the misrepresentation need not induce the victim to give up her property. Correct B For larceny by false pretenses, the victim intends to convey title, but for larceny by trick, the victim intends to convey only custody. C For larceny by false pretenses, the victim intends to convey only possession, but for larceny by trick, the victim intends to convey title. D For larceny by false pretenses, the misrepresentation need not induce the victim to give up her property, but for larceny by trick, the victim must actually rely on the misrepresentation in giving up her property.

False pretenses differs from larceny by trick in that title is obtained through false pretenses, but only custody of the property is obtained through larceny by trick. What is obtained depends upon what the victim intended to convey to the defendant. For both crimes, the victim must actually be deceived by, or act in reliance on, the misrepresentation, and this must be a major factor (or the sole cause) of the victim passing title (larceny by false pretenses) or custody (larceny by trick) to the defendant.

Under the plain view exception to the warrant requirement, which of the following is not required? response - correct A The item must be evidence, contraband, or a fruit or instrumentality of a crime. Correct B The police must have inadvertently discovered the item. C The police must be legitimately on the premises where the item is found. D It must be immediately apparent that the item is evidence, contraband, or a fruit or instrumentality of a crime.

For the warrantless seizure of an item under the plain view exception, it is not required that the item be inadvertently discovered. Formerly, this exception to the warrant requirement applied only if the item was inadvertently discovered. Inadvertence, however, is no longer a requirement. To make a warrantless seizure, the police (1) must be legitimately on the premises where the item is found; (2) the item must be evidence, contraband, or a fruit or instrumentality of a crime; (3) the item must be in plain view; and (4) it must be immediately apparent (i.e., probable cause) that the item is evidence, contraband, or a fruit or instrumentality of a crime.

An employee of the state government always received his state paycheck on the last workday of the month. The employee was not a good money manager, and just barely managed to make it from paycheck to paycheck each month. On the second to the last workday of the month, the employee had $45 in his checking account, and, needing to buy a birthday gift for his sister, he wrote a check to a gift boutique for $100. He knew that he would be receiving his paycheck the next day, so he could deposit the paycheck before the check would be sent to the bank. However, unbeknownst to the employee, the state legislature was having a budget impasse. Because the state constitution prohibited any deficit spending, state employees were not paid as usual. Without a paycheck to deposit, the check written to the gift boutique was returned for insufficient funds. The merchant complained to the police, who arrested the employee and charged him under a statute that prohibited "issuing a check knowing that it is drawn against insufficient funds, with intent to defraud the payee of the check." What should be the outcome of the employee's prosecution? response - correct Correct ANot guilty, because the employee intended to deposit his paycheck the next day. BNot guilty, because it was reasonable for the employee to expect that he would receive his paycheck as usual. CGuilty, because the employee knew when he wrote the check that he did not have sufficient funds in his account to honor it. DGuilty, because reliance on a future source of income does not vitiate the employee's violation of the statute when he wrote the check.

Given that the employee intended to deposit his paycheck before the checks cleared, he lacked the intent to defraud required by the statute. The statute under which the employee is being prosecuted is a variation of the offense of false pretenses. As with false pretenses, the statute requires a specific intent, i.e., an intent to defraud. If the employee intended to deposit sufficient funds to honor the check before it reached his bank, then the employee did not intend to defraud the gift boutique. Thus, the employee lacked the specific intent that is a necessary element of the crime charged. (B) is incorrect because the employee's expectation that he would receive his paycheck as usual need not have been reasonable. Even if such an expectation were unreasonable, the employee would not be guilty if he did not intend to defraud the payees, as required by the statute. (C) is incorrect because it would result in a verdict of guilty without requiring intent to defraud. Knowledge that the check was drawn against insufficient funds is just one element of the statute. The intent to defraud is also required to convict under the applicable statute. (D) also incorrectly assumes that the employee violated the statute merely by knowingly writing a check on insufficient funds. As explained above, the requisite intent to defraud is absent. Thus, there is no "violation" to be vitiated.

Which of the following is most likely to be found to be a seizure for purposes of the Fourth Amendment? response - correct A A police officer approaches a suspect, orders the suspect to stop, and the suspect runs. Correct B A police officer approaches a suspect and draws her revolver, orders the suspect to stop, and the suspect complies. C A police officer turns on his squad car's overhead lights as the squad approaches a suspect, and the suspect runs. D A police officer boards a bus, asks a suspect for identification and consent to search his luggage, and the suspect agrees.

If a police officer approaches a suspect and draws her revolver, orders the suspect to stop, and the suspect complies, this will most likely be found to constitute a seizure. For purposes of the Fourth Amendment, a seizure occurs when a reasonable person would believe that he is not free to leave. The courts consider the totality of the circumstances in making this determination. If a police officer turns on his squad car's overhead lights as the squad approaches a suspect and the suspect runs, a court will not likely find that there was a seizure. A seizure requires a physical application of force (e.g., handcuffing or otherwise subduing a person) or submission to an officer's show of force. If a police officer approaches a suspect, orders the suspect to stop, and the suspect runs, there is no application of force and no submission to the show of force, which would require, at the least, stopping in response to the officer's order. Therefore, it is unlikely that a court would find a seizure under the Fourth Amendment in these circumstances. If a police officer boards a bus, asks a suspect for identification and consent to search his luggage, and the suspect agrees, a court would not likely find a seizure for Fourth Amendment purposes. Police officers may ask people for permission to search and for identification; such requests do not involve the physical application of force or submission to a show of force.

Which of the following statements regarding embezzlement is true? response - correct A The defendant must take and carry away the property. Correct B If the defendant intends to restore the exact property taken, it is not embezzlement. C The misappropriation of property occurs at the time the defendant obtains wrongful possession of the property. D The conversion must result in direct personal gain to the defendant.

If the defendant intended to restore the exact property taken, it is NOT embezzlement. But if he intended to restore similar or substantially identical property, it is embezzlement, even if it was money that was initially taken and other money-of identical value-that he intended to return. In embezzlement, the misappropriation of the property occurs while the defendant has lawful, NOT wrongful, possession of it. On the other hand, in larceny, it occurs generally at the time the defendant obtains wrongful possession of the property. In embezzlement, since the defendant is already in possession of the property, there is NO need that he take and carry away the property. Rather, embezzlement requires a fraudulent conversion of the property. The conversion need NOT result in direct personal gain to the defendant. The conversion required by embezzlement requires only that the defendant deal with the property in a manner inconsistent with the trust arrangement pursuant to which he holds it.

Assuming all other elements are met, in which of the following circumstances would the defendant most likely not be convicted of homicide? response - correct A The defendant stabs the victim in the heart; at exactly the same time, another person shoots the victim in the head. Either act would have independently caused the victim's death, but it is unclear which act caused the victim's death. Correct B The defendant stabs the victim but does not kill him; due to construction at the hospital where the victim is admitted for treatment of the stab wound, the victim contracts asbestosis and dies from it one year later. C The defendant stabs and kills a victim who has terminal cancer. D The defendant stabs the victim, who has hemophilia; the victim bleeds to death as a result of the stabbing.

If the defendant stabs the victim but does not kill him, and due to construction at the hospital where the victim is admitted for treatment of the stab wound, the victim contracts asbestosis and dies as a result one year later, the defendant is not likely to be convicted of homicide. To be convicted of homicide, the defendant must have actually and proximately caused the death of the victim. An intervening act that presents a foreseeable risk will generally not break the chain of causation. However, an unforeseeable risk, such as an injury due to hospital construction, will most likely break the chain of causation. Stabbing and killing someone with terminal cancer is likely to be considered homicide. A defendant may be guilty of killing a victim who was going to die anyhow. Actual and proximate causation is not broken if the defendant ends the victim's life prematurely, even by a short time. If a defendant stabs a victim while another person shoots the victim, the defendant is likely to be found guilty of homicide. Simultaneous acts by multiple persons may be considered independent, sufficient causes of a single result. Thus, multiple persons may be convicted of homicide even though there was only one death. A defendant who stabs a victim with hemophilia who bleeds to death is likely to be found guilty of homicide. Any preexisting conditions that make a person more susceptible to death are essentially disregarded. The defendant "takes the victim as he finds him."

____________ is an element of __________. response - correct Correct A Intent to defraud; false pretenses B Obtaining title; larceny by trick C Trespass; false pretenses D Trespass; embezzlement

Intent to defraud is an element of false pretenses. The offense of false pretenses generally consists of: (i) obtaining title; (ii) to the property of another; (iii) by an intentional (or, in some states, knowing) false statement of past or existing fact; (iv) with intent to defraud the other. Obtaining title is not an element of larceny by trick; rather with larceny by trick, the defendant merely obtains possession, not title. Trespass is not an element of false pretenses or embezzlement. Embezzlement is the fraudulent conversion of the property of another by a person in lawful possession of the property.

Jeopardy attaches in a jury trial when: response - correct A Opening statements begin Correct B The jury is empaneled and sworn C The first witness is sworn D The judge delivers jury instructions

Jeopardy attaches in a jury trial when the jury is empaneled and sworn. Under the Fifth Amendment right to be free of double jeopardy, a defendant may not be retried for the same offense once jeopardy has attached. Jeopardy does not attach in a jury trial when the first witness is sworn. This is when jeopardy attaches in a bench trial, not in a jury trial. By the time opening statements begin or the judge delivers jury instructions, jeopardy has already attached.

Which of the following are specific intent crimes? response - correct A False imprisonment, kidnapping, and battery Correct B Solicitation, assault, and burglary C Assault, battery, and first degree murder D Larceny, robbery, and arson

Solicitation, assault (attempted battery type) and burglary are specific intent crimes. Solicitation requires the intent to have the person solicited commit the crime. The attempted battery type of assault, like all attempt crimes, requires the specific intent to commit the crime attempted. Burglary requires the specific intent to commit a felony at the time of entry into the dwelling of another. Assault (attempted battery type) is a specific intent crime. First degree murder is a specific intent crime, in that it requires the intent to kill after premeditation and deliberation. However, battery is a general intent crime. False imprisonment, kidnapping, and battery are all general intent crimes. Larceny and robbery are both specific intent crimes that require the intent to permanently deprive another person of his interest in the property taken. However, arson is a malice crime.

Miranda warnings are required as a prerequisite to the admissibility of confessions resulting from custodial police interrogation. Which of the following statements is true about the interrogation requirement under Miranda? response - correct A Spontaneous statements violate Miranda if made before Miranda warnings are given. Correct B The term "interrogation" includes any police tactic that officers should know is likely to elicit an incriminating response. C Allowing a suspect to talk to his wife in the presence of the police can constitute interrogation. D Routine booking questions usually will be considered interrogation.

The term "interrogation" includes any police tactic that officers should know is likely to elicit an incriminating response. It is not limited to direct questioning. The Supreme Court has held that for purposes of Miranda, allowing a suspect to talk to his wife in the presence of the police DOES NOT constitute interrogation. The Supreme Court has also held that routine booking questions do not constitute interrogation (because information about a suspect's name or address usually is not incriminating). Finally, if a spontaneous statement is made before Miranda warnings are given, there is no Miranda violation. Spontaneous statements are statements not made in response to any police conduct. Where the suspect blurts out information upon seeing the police, there is no interrogation.

Which of the following statements is true regarding a detainee's Fifth Amendment right to counsel under Miranda? response - correct A All doubts about a request for counsel are construed in favor of the detainee. Correct B The right applies only at custodial interrogations by the police or one known to be an agent of the police. C The police may question the detainee about an unrelated crime if they scrupulously honor the request by waiting a few hours and rewarning the defendant before the new questioning begins. D A detainee cannot waive the right to counsel in the absence of counsel.

The Fifth Amendment right to counsel under Miranda applies only to custodial interrogations by the police or one known to be an agent of the police. The purpose of the rule is to protect the right against self-incrimination by preventing the police from badgering a suspect until he talks. The warnings are intended to offset the coercive atmosphere of custodial police interrogation, and so the right does not apply when the suspect is not in custody or is not being questioned by a police officer or one known by the suspect to be an agent of the police. If a detainee invokes his Miranda right to counsel, the police may NOT question him about an unrelated crime. This is different from the rule for when the detainee merely invokes his right to remain silent, in which instance the police may question about a different crime if they scrupulously honor the request. It is not true that all doubts about a request for counsel are construed in favor of the detainee. A request for counsel must be unambiguous and specific. Neither is it true that a detainee cannot waive the right to counsel in the absence of counsel. A waiver will be valid if it was knowing, voluntary, and intelligent.

The Sixth Amendment right to counsel applies __________. response - correct A only at interrogations performed by one known to be a government agent Correct B only after adversary judicial proceedings have begun C whenever an informant is placed in the defendant's cell D whenever a defendant gives a blood sample

The Sixth Amendment right to counsel applies only after adversary judicial proceedings have begun (e.g., formal charges have been filed). It is not true that the Sixth Amendment applies whenever an informant is placed in the defendant's cell. It applies when an informant is placed in a defendant's cell after adversary judicial proceedings have been initiated. But there is no Sixth Amendment violation just because an informant is placed in the defendant's cell after charges are filed. There is a violation only if the informant does something designed to elicit incriminating remarks. The choice providing that the Sixth Amendment applies only at interrogations performed by one known to be a government agent is too limiting to be correct. A limitation like this one applies to Miranda warnings. The warnings must be given before interrogation by one known to be a government agent in order to offset the coercive nature of police interrogation. A similar rule does not apply under the Sixth Amendment right to counsel. The Sixth Amendment right to counsel applies to all post-charge interrogations, whether or not the defendant knows he is speaking to a government informant. Finally, the blood sample choice is incorrect. The Sixth Amendment right to counsel applies at critical stages of a criminal prosecution after formal proceedings have begun, and the taking of a blood sample is not a critical stage.

Although government-required drug testing constitutes a search, the Supreme Court has upheld such testing without a warrant, probable cause, or even individualized suspicion when justified by "special needs" beyond a general interest of law enforcement. In which of the following cases is a court least likely to find a special need justifying a warrantless drug test? response - correct A Railroad employees involved in accidents. Correct B Politicians running for public office. C Public school students participating in extracurricular activities. D Drug interdiction agents who have access to large quantities of illegal drugs.

The Supreme Court found no special interest justifying the warrantless drug testing of politicians running for public office. The Supreme Court has found a special interest justifying warrantless drug testing of public school students participating in extracurricular activities-the need being to assure the safety of students. The Court has also found a special interest for warrantless drug testing of railroad employees involved in accidents-the strong interest of assuring public safety. Finally, the Court has found a special interest justifying the warrantless drug testing of drug interdiction agents-their ready access to drugs.

For Fourth Amendment purposes, which of the following people is least likely to be found to have a reasonable expectation of privacy in the place searched? response - correct A A person who lives in the premises that were searched but does not own it. Correct B A person who came to the premises that were searched to buy illegal drugs. C A person who owns the premises that were searched but does not live there. D A person who was an overnight guest at the place searched.

The Supreme Court has held that a person who was on the premises that were searched to buy illegal drugs does not have a reasonable expectation of privacy in the premises. A claim that a search violated the Fourth Amendment can be raised only by a person who has a reasonable expectation of privacy in the place searched. The Supreme Court has held that a person has a reasonable expectation of privacy any time she owns the place that was searched or has a right to possession of it; whether or not she lives there would not affect this interest. The Court has also held that if the place searched is the person's home, then she has a reasonable expectation of privacy, regardless if she owned or had a right to possess it. Finally, the Supreme Court has held that an overnight guest at the place searched also has a reasonable expectation of privacy in the premises for Fourth Amendment purposes.

A husband who believed that his wife was having an affair with his brother hired an arsonist to burn down the brother's house. They planned for the husband to take his brother to a ballgame so that the arsonist would be able to set the house on fire without detection. After the husband and brother left for the ballgame, however, the arsonist decided to abandon the plan and immediately left town without doing anything further. When the husband returned from the ballgame with the brother, he saw the house still standing and blurted out what was supposed to have happened. The husband and the arsonist were arrested and charged with conspiracy to commit arson. At the arsonist's trial, his attorney argued that he was innocent of the conspiracy because he decided not to go ahead with the plan, and nothing criminal had in fact occurred. At common law, how should a jury find the arsonist? response - correct ANot guilty of conspiracy, because going to a ballgame is not a criminal overt act. BNot guilty of conspiracy, because the husband, not the arsonist, committed the overt act. CGuilty, because the husband executed his part of the plan. Correct DGuilty, because the arsonist agreed to set the brother's house on fire.

The arsonist should be found guilty. A conspiracy is a combination or agreement between two or more persons to accomplish some criminal or unlawful purpose, or to accomplish a lawful act by unlawful means. The mens rea required for conspiracy is specific intent, in that both parties must intend to agree to accomplish some criminal or unlawful purpose. Thus, once the arsonist was hired by the husband and they came up with a plan to burn down the brother's house, the crime of conspiracy was completed. (C) is incorrect because it implies that carrying out the plan by at least one party is required; the conspiracy was complete even before the husband fulfilled his duties under the plan. Note that, while most states now require an overt act for conspiracy, the common law version does not. (A) is incorrect. Even if an overt act were required, it need not be in and of itself criminal. (B) is also incorrect. If an overt act were required, it need only be performed by one of the co-conspirators, not necessarily the conspirator on trial.

Three thieves agreed to rob a bank. The first was to steal a car to be used for the getaway, the second agreed to procure weapons, and the third would check the bank for cameras. The car thief stole a car and parked it in a lot behind his girlfriend's apartment building. While visiting her the night before the robbery, the car thief suffered a series of convulsive seizures. He was rushed to the hospital where he was placed in the intensive care unit and heavily sedated. Meanwhile the two other thieves, unaware of their accomplice's illness, met and decided to rob the bank on their own, despite the absence of a getaway car and driver. They robbed the bank, but were quickly apprehended as they tried to escape and implicated the car thief under police questioning. The car thief can be charged with: response - correct ATheft of the car only. BConspiracy to commit robbery and theft of the car only. CRobbery and theft of the car only. Correct DTheft of the car, conspiracy to commit robbery, and robbery.

The car thief can be charged with theft, conspiracy, and robbery. The facts clearly show that the car thief is guilty of the theft of the car that was to be used for the getaway. Conspiracy requires: (i) an agreement between two or more persons; (ii) the intent to enter into the agreement; and (iii) the intent to achieve the objective of the agreement. Most states also require an overt act in furtherance of the conspiracy. The car thief conspired to commit robbery, because he entered into an agreement with the two other thieves to rob the bank, intending both to enter into such an agreement and to achieve the objective thereof. The car thief's theft of the car constituted an overt act in furtherance of the conspiracy. Thus, the car thief can be charged with conspiracy. Furthermore, each member of a conspiracy is liable for the crimes of all other conspirators if: (i) such crimes were committed in furtherance of the objectives of the conspiracy; and (ii) such crimes were a natural and probable consequence of the conspiracy. A conspirator may limit his liability for subsequent acts of the other members of the conspiracy if he withdraws from the conspiracy by performing an affirmative act that notifies all members of the conspiracy in time for them to have the opportunity to abandon their plans. The car thief's absence from the robbery scene with the getaway car was due to his sudden illness, rather than any voluntary decision on his part to withdraw from the conspiracy. Thus, the car thief failed to make a legally effective withdrawal from the conspiracy. Certainly, the robbery of the bank, which was the sole object of the conspiracy, was a crime committed in furtherance of the conspiracy's objectives and was a natural and probable consequence of the conspiracy. Therefore, the car thief is liable for this robbery committed by his co-conspirators. Because the car thief can properly be charged with all three crimes, (A), (B), and (C) are incorrect.

The statutes of a state define the following crimes (with the most serious listed first): First degree murder-Premeditated or intentional killing. Felony murder-Killing while in the act of committing a common law felony. Second degree murder-Killing with reckless disregard for the safety of others. Manslaughter-Killing with adequate provocation or through criminal negligence. A competitive camp counselor who was determined to have her team win a relay race decided to put a colorless and odorless drug into the other teams' water bottles. The counselor wanted the other teams to become sick to their stomachs so that they could not run as fast in the race. The counselor knew that people could become very ill, or even die, if they consumed too large a quantity of the drug but she only intended to place a small amount in each water bottle. The day before the race, the counselor went into the nurse's office and took a bottle of the drug. The next day she woke up early and went to the cafeteria to put small quantities of the drug into the other teams' water bottles. She unintentionally put a large amount in a few of the water bottles. Several campers became extremely ill and one eventually died. What is the most serious crime for which the counselor may be convicted? response - correct AFirst degree murder. BFelony murder. Correct CSecond degree murder. DManslaughter.

The counselor may be convicted of second degree murder. Under the statute provided in the question, second degree murder is a killing committed with a reckless disregard for the safety of others. Here, the counselor consciously disregarded a substantial and unjustifiable risk that the drug she was placing in the water bottles could be seriously harmful or even fatal. Thus, the counselor may be convicted of second degree murder. (A) is wrong. The counselor could not be convicted of first degree murder because she lacked the necessary intent to kill; the counselor's only intent was to make the other team's members sick. (B) is wrong. The counselor cannot be convicted of felony murder because she did not commit a felony; a majority of jurisdictions would consider simple battery to be a misdemeanor. Furthermore, the felony generally must be independent of the killing. A battery (or aggravated battery) would not be considered independent of the conduct which kills (although it might be considered to be a valid basis for a manslaughter conviction). (D) is not as good an answer as (C). Although a jury could find that her actions were criminally negligent, resulting in manslaughter, a jury could also find that she acted with reckless disregard, resulting in second degree murder. Thus, given that the call of the question asks for the most serious crime for which the counselor could be convicted, (C) is correct.

The defendant's neighbor owned an authentic major league baseball signed by Babe Ruth. The defendant asked if he could show it to some friends who were visiting. The neighbor agreed as long as he kept it in the display case, which the defendant promised to do. In fact, the defendant intended to use the ball in a pickup game. During the game, the ball was hit over the fence and into a yard with a guard dog, which had chewed up several other balls that had previously landed in the yard. The dog did the same to that ball. When the neighbor learned what happened to the ball, he pressed charges against the defendant. If the defendant is convicted, he will most likely be found guilty of what crime? response - correct ACommon law larceny. BEmbezzlement. CFalse pretenses. Correct DLarceny by trick.

The defendant is guilty of larceny by trick because he obtained possession of the baseball by means of a misrepresentation. Larceny is the taking and carrying away of tangible personal property of another by trespass, with intent to permanently (or for an unreasonable time) deprive the person of her interest in the property. The taking must be without the consent of the person in possession of the property. If such consent is induced by a misrepresentation of a past or existing fact, the consent is not valid. The resulting larceny is called larceny by trick. Here, the defendant obtained possession of the baseball with the owner's consent. However, this consent was obtained by means of the defendant's misrepresentation about friends visiting. This was a false statement of an existing fact, made with the intent that his neighbor rely on the statement, and the misrepresentation induced his neighbor's consent. At the time of this taking, the defendant intended to deal with the baseball in a manner that involved a substantial risk of damage or loss. This suffices as intent to permanently deprive. Therefore, all the elements are in place for larceny by trick. (A) is not as good a choice as (D) because the taking in this case is better characterized as larceny by trick rather than larceny, given that the defendant induced his neighbor to consent to his taking possession of the baseball. (C) is incorrect because the defendant obtained only possession of the baseball, not title. False pretenses differs from larceny by trick in what is obtained. If the defendant obtains only possession of the property, the offense is larceny by trick, whereas obtaining of title means that false pretenses has been committed. What the victim intended to convey to the defendant is determinative. The neighbor intended only to let the defendant borrow the baseball for a short time, not to convey title to him. Consequently, the only thing the defendant obtained was possession of the baseball. Because title to the baseball was not obtained, there can be no conviction of false pretenses. Regarding (B), embezzlement is the fraudulent conversion of property of another by a person in lawful possession of that property. In embezzlement, misappropriation occurs while the defendant has lawful possession of the property, while in larceny, it occurs generally at the time the defendant obtains wrongful possession of the property. Here, as detailed above, the defendant's taking of possession of the baseball was trespassory due to the manner in which he obtained consent to such possession. The crime of larceny was complete on the defendant's taking possession with the requisite intent to permanently deprive. Thus, at the time the baseball was destroyed, the defendant had already misappropriated it and was not in lawful possession of it. As a result, there can be no conviction for embezzlement.

A thief was passing by a house under construction when he noticed that the ladder being used by workers on the roof had copper braces supporting the rungs. After making sure that the workers on the roof could not see him, the thief used pliers that he had in his pocket to remove all of the copper braces that he could reach from the ground. A short time later, a worker climbed down the ladder and it collapsed. He fell to the ground and severely injured his back. The thief was apprehended a few hours later trying to sell the copper for scrap. A statute in the jurisdiction makes it a felony for "maliciously causing serious physical injury to another." The thief was charged with malicious injury under the statute and was also charged with larceny. After a jury trial in which the above facts were presented, he was convicted of both charges. If he appeals the conviction for the malicious injury charge on grounds of insufficient evidence, how should the court rule? response - correct AAffirm the conviction, because the thief was engaged in criminal conduct at the time of the act that resulted in the injury. Correct BAffirm the conviction, because the jury could have found that the thief acted with malice. CReverse the conviction, because there was no evidence that the thief intended to injure anyone. DReverse the conviction, because there was no evidence that the thief bore any malice towards the workers on the roof.

The court should affirm the thief's conviction. Crimes imposing a mens rea of malice generally do not require the proof of intent that specific intent crimes require. It is sufficient if the defendant recklessly disregarded an obvious or high risk that the particular harmful result would occur. Here, the facts presented were sufficient to allow the jury to conclude that the thief knew of the probability that the ladder would collapse without the braces when someone climbed down it, and acted in reckless disregard of that risk by removing the braces. (A) is incorrect because the fact that the thief was committing larceny when he removed the braces does not establish malice for purposes of the malicious injury charge. Even if his conduct were otherwise legal, he could be liable for that charge if he acted with reckless disregard of the high risk of injury. (C) is incorrect because, as discussed above, it is generally not necessary to show an intent to injure for a crime requiring a mens rea of malice; reckless disregard of an obvious risk will usually suffice. (D) is incorrect because crimes requiring a mens rea of malice do not refer to malice in the dictionary sense; a showing of ill will or hatred of the victim is not required.

Acting on a hunch, a police officer went to a young woman's apartment, broke in, and searched it. The officer found exactly what she was looking for under the woman's bed: a sack filled with jewels. The attached note read, "Sweetheart, here are the goods from the estate heist. Your loving boyfriend." It was well known in the community that the woman's boyfriend was a jewel thief. The officer also knew that the estate of a local socialite had been burglarized three days ago. Just as the officer finished reading the note, the woman returned. The officer immediately placed the woman under arrest as an accessory to the estate burglary. Based on the evidence obtained from the woman's apartment, a search warrant was issued for her boyfriend's apartment. The search yielded burglar tools and more jewels from the estate. The boyfriend was immediately arrested and charged with the estate burglary. At the boyfriend's trial for the estate burglary, his attorney files a motion to suppress the evidence consisting of the bag of jewels and note, the tools, and the jewels from the boyfriend's apartment. How should the court rule on the motion? response - correct AGrant the motion as to the bag of jewels and note, but deny it as to the evidence found in the boyfriend's apartment. BGrant the motion, because all of this evidence is fruit of the poisonous tree. CDeny the motion, because the police would have caught the boyfriend with the goods eventually. Correct DDeny the motion, because the police had a warrant to search the boyfriend's apartment.

The court should deny the motion to suppress because the police had a warrant to search the boyfriend's home. The boyfriend's expectation of privacy extended only to his own home, which was searched under a warrant. He does not have standing to assert a Fourth Amendment claim regarding the search of his girlfriend's apartment because her apartment was not his home, and he did not own it or have a right to possession of it. Thus, (A) is incorrect. Because the boyfriend cannot object to the search that provided the probable cause for the search of his apartment, (B) is also incorrect. (C) is not a valid justification because there is nothing to indicate that the seizure would fall under the "inevitable discovery" exception to the exclusionary rule.

The police obtained a valid arrest warrant for a drug dealer. A reliable informant told the police that the drug dealer was staying at a friend's house until "the heat was off." Without having obtained a search warrant, the police went to the friend's house, knocked on the door, and asked the friend if the drug dealer was there. The friend replied that the drug dealer had been staying at the house for a few days but had left a few hours ago. The police pushed open the door and began searching for the drug dealer. They found him hiding in a closet along with two five-pound bricks of marijuana. They arrested both the drug dealer and the friend. Before his trial for possession of marijuana, the friend moved to suppress the marijuana found in the closet. Should the court grant the motion to suppress? response - correct Correct AYes, because a search warrant was required. BYes, because the police may not execute an arrest warrant at the third party's home. CNo, because the police had probable cause to believe that the drug dealer was staying at the friend's home. DNo, because the police had a valid arrest warrant and the marijuana was found incident to the arrest.

The court should grant the motion to suppress because a search warrant was required. Absent exigent circumstances, the police executing an arrest warrant may not search for the subject of the warrant in the home of a third party without first obtaining a separate search warrant for the home. If the police do execute an arrest warrant at the home of a third party without obtaining a search warrant for the home, the arrest is still valid but evidence of any crime found in the home cannot be used against the owner of the home because it is the fruit of an unconstitutional search. Thus, (A) is correct and (D) is incorrect. (B) is incorrect because it is too broad. A person can be arrested at the home of a third party, but the police generally cannot enter the third party's home without consent unless they have a search warrant for the home. (C) is incorrect because, as discussed above, a search warrant is required absent exigent circumstances, which are not present in this case. Here, the probable cause established by the informant's disclosure would have enabled the police to obtain a search warrant.

****A police officer learned from a reliable informant that a major drug deal was about to take place at a local restaurant. The officer obtained a search warrant for the restaurant and arrived with other uniformed officers to search the premises. While conducting the search, the officer searched several of the customers. While searching one of the restaurant's regular customers, the officer felt an object in the customer's pocket and pulled out a container filled with heroin. The customer was arrested and later convicted of possession of heroin. A state statute permits officers executing a search warrant to search persons on the premises if the officers reasonably expect danger to themselves or a risk of disposal or concealment of anything described in the warrant. If the customer challenges his conviction on the ground that his Fourth Amendment rights were violated, will he be successful? response - correct AYes, because the statute is vague and overbroad. Correct BYes, because his presence in the place to be searched by the police does not negate the requirement of probable cause. CNo, because the search was conducted pursuant to a valid search warrant. DNo, because the search was authorized by statute.

The customer will be successful. To be reasonable under the Fourth Amendment, most searches must be pursuant to a warrant. The warrant must describe with reasonable precision the place to be searched and the items to be seized. A search warrant does not authorize the police to search persons found on the premises who are not named in the warrant. In Ybarra v. Illinois (1979), a case based on similar facts, the Supreme Court held that "each patron of the tavern had an individual right to be free of unreasonable searches, and presence at a location subject to search does not negate the requirement of probable cause to search the person present." (A) is incorrect because the validity of the statute is not the primary issue. Even in the absence of a statute, the search of the customer by the officer violated the customer's Fourth Amendment rights. (C) is incorrect because, as discussed above, the search warrant did not override the customer's Fourth Amendment rights. While the police would be able to search a person discovered on the premises for whom they had probable cause to arrest, because the search would be incident to a lawful arrest, here they searched the customer prior to an arrest and without probable cause. (D) is irrelevant; if a search is unconstitutional, it does not matter that it was authorized by statute. To the extent that the statute authorizes a search in violation of the Fourth Amendment, it is unconstitutional.

Acting pursuant to a valid search warrant, the police entered and searched the defendant's garage and discovered a cardboard box containing cocaine in the rafters storage area. The box was securely taped and bore a freight label addressed to the defendant's friend. At his trial for violation of the jurisdiction's statute making it a felony to knowingly possess cocaine, the defendant testified that his friend had brought him the package a week before it was seized by the police, telling him that he needed to store it in the defendant's garage. The defendant also testified that he had not asked the friend what it contained. What additional facts must the prosecution prove to establish the defendant's liability for the charged felony? response - correct AThat he knew or believed that the box contained cocaine and had moved or handled the box. Correct BThat he knew or believed that the box contained cocaine. CThat he should have known that the box contained cocaine and had moved or handled the box. DNo additional facts.

The defendant should be found guilty of the charged felony if he knew or believed that the box contained cocaine. The defendant is being tried for "knowingly" possessing cocaine. A person does not act knowingly unless he is aware that his conduct is of the proscribed nature or that the proscribed circumstances exist. Thus, the defendant could not have acted knowingly unless he knew or believed that the box contained cocaine. (A) is incorrect because criminal statutes that penalize the possession of contraband generally require only that the defendant have control of the item for a long enough period to have had an opportunity to terminate the possession. Thus, the defendant need not have moved or handled the box. (C) is incorrect for the same reason as (A), and also because the defendant's failure to know when he should have known would constitute negligence-failure to be aware of a substantial risk that prohibited results will follow or that circumstances exist-and negligence is not sufficient to establish knowledge. Note, however, that a defendant may not consciously avoid learning the true nature of the item possessed; knowledge may be inferred when the defendant is aware of a high probability of the true nature of the item and deliberately avoids learning the truth. (D) is incorrect because, as discussed above, the statute requires that the defendant knew or believed that the box contained cocaine.

The defendant and the victim got into a minor verbal altercation, concluding with the defendant lightly shoving the victim. The victim lost his balance and struck his head on the pavement, causing serious bodily injury. The defendant was charged with battery, which is defined in the jurisdiction as "purposely or knowingly causing serious bodily injury to another." Should the defendant be convicted of battery? response - correct Correct ANo, because the defendant did not know that the victim would be seriously injured. BNo, because the defendant did not strike a serious blow to the victim. CYes, because the defendant purposely shoved the victim. DYes, because the victim suffered serious bodily injury.

The defendant should not be convicted of battery. Under the statute's fault standards, a defendant must have acted purposely (i.e., with conscious intent to cause the result) or knowingly (i.e., with knowledge that his conduct will necessarily or very likely cause the result) as to the harmful result. The apparent inference to be drawn from the facts is that the defendant did not consciously desire, nor contemplate to a practical certainty, the serious injury to the victim that actually occurred. Had the defendant intended to cause such severe harm, he no doubt would have dealt the victim a strong blow rather than simply giving the victim a light shove. Therefore, as to the nature of the result, the defendant did not act with "purpose" or "knowledge" as those terms are defined in the Model Penal Code and modern criminal codes. (B), while close, is not as good an answer as (A) because it does not address the state of mind issue in the problem. A light shove might be sufficient for a battery as defined under a different set of facts (e.g., if the defendant believes that the victim would fall down stairs with a light shove). (C) is incorrect because it addresses the act but not the result. As defined in this question, battery must not only be committed by a purposeful act, but also be done with a "purposeful" or "knowing" state of mind as to the result. (D) is incorrect for much of the same reason-the state of mind requirement also applies to the result, as discussed above. The injury must have been purposely or knowingly caused, and that concept is not contained within choice (D).

The victim owned a cottage in an ocean resort area. He stayed there only during the summer months, and left the cottage unoccupied during the balance of the year. The defendant, a resident of a neighboring cottage, was aware of this practice. For a change in his routine, however, the victim decided to spend a week at the cabin in the off-season. Unaware that the victim was occupying the cottage, the defendant decided to borrow a portable television set that he knew the victim kept in the cottage. To avoid being seen, he entered the cottage late at night, using a key under the front doormat. He found the television set, disconnected it, and headed for the rear of the house to leave. He opened the kitchen door and found the victim seated there in the dark, having a late night snack. Both men were startled and neither man recognized the other in the dark. The defendant assumed that the victim was a burglar, and was afraid that he might be armed. Trying to flee the kitchen as quickly as possible, the defendant dropped the television set in the middle of the kitchen floor. As the set hit the floor, the picture tube exploded with a loud noise. The noise so frightened the victim that he had an immediate heart attack and died. If the defendant is charged with felony murder as the result of the victim's death, what is his best defense? response - incorrect AHe did not intend to kill the victim. Correct BHis only intent was to borrow the television set for a few days. CLarceny is not an inherently dangerous crime, and it was not being committed in an inherently dangerous manner. Incorrect DThe victim's heart attack was an unforeseeable consequence of the defendant's acts.

The defendant's best defense to felony murder is that he only intended to borrow the television set for a few days. By establishing this intent, the defendant will show that he did not have the intent to commit a felony and therefore cannot be guilty of felony murder. A killing (even if accidental) committed during the course of a felony is murder. Malice is implied from the intent to commit the underlying felony. To convict a defendant of felony murder, the prosecution must prove that he committed or attempted to commit the underlying felony. Here, the possible felonies being committed by the defendant, during which the victim's death occurred, would be larceny and burglary. Burglary requires the intent to commit a felony within the dwelling, and larceny requires the intent to permanently deprive a person of his interest in property. If the defendant's only intent was to borrow the victim's television set for a few days, then the defendant lacked the intent to permanently deprive the victim of his interest in the set; i.e., the requisite intent for larceny is missing. Likewise, the absence of intent to steal the set would mean that, at the time of breaking and entering the cottage, the defendant did not intend to commit a felony therein. Consequently, the defendant is not guilty of burglary. Because under these circumstances no felony would have been attempted or committed, it cannot be shown that the death of the victim occurred during the commission of a felony. Therefore, the defendant would not be guilty of felony murder. (D) is tempting, because generally a conviction of felony murder requires that the death must have been a foreseeable result of commission of the felony. However, some courts do not apply a foreseeability requirement and require only that the underlying felony be malum in se. Furthermore, even those courts applying a foreseeability requirement have been willing to find most deaths occurring during the commission of a felony to be foreseeable. Here, the defendant believed that the cottage was unoccupied for the winter. Thus, it was arguably unforeseeable that the defendant's entering the cottage and taking a television set would result in the death of an occupant, but it is by no means certain that a court would agree. Furthermore, in those jurisdictions that do not require foreseeability of death, the defendant could be convicted of felony murder if the death occurred during the commission of a burglary, because burglary is always classified as a malum in se felony. Because the circumstances in choice (B) would assure the defendant of avoiding conviction in all jurisdictions, (B) is a better answer than (D). (A) incorrectly focuses on intent to kill. Intent to kill is one of the states of mind by which a defendant is deemed to have malice aforethought, which is necessary for a killing to constitute murder. However, this question refers to felony murder, wherein malice aforethought exists in the form of intent to commit a felony. Thus, it is irrelevant whether the defendant intended to kill the victim. Regarding (C), it is true that most courts limit the felony murder doctrine to felonies that are inherently dangerous, and that larceny generally is not considered to be inherently dangerous. However, assuming the existence of the requisite intent, the defendant may have committed burglary, which is deemed to be inherently dangerous. Thus, (C) might provide no defense at all to a charge of felony murder.

At the defendant's prosecution for robbery of a drugstore, the main prosecution witness testified that the defendant had asked her to drive him to the town where the drugstore was located. The witness testified that the defendant did not explain his purpose for going to the town, and that he had stopped at a relative's house along the way to pick up a bundle that could have been the sawed-off shotgun used by the robber. On cross-examination, the defendant's attorney asked a number of pointed questions of the witness, implying that the defendant had asked her to drive to the town so that he could visit relatives there and suggesting that the witness had obtained a sawed-off shotgun for use by a confederate. The defendant did not testify on his own behalf. In final argument, the prosecutor called the jury's attention to the two versions of events suggested by the witness's testimony on direct examination and the defense attorney's questions on cross-examination, and then said, "Remember, you only heard one of the two people testify who know what really happened that day." If the defendant is convicted of robbery, will his conviction likely be upheld? response - correct Correct ANo, because the prosecutor's comment referred to the defendant's failure to testify, a violation of his Fifth Amendment privilege of silence. BNo, because under the circumstances the attack on the witness's credibility was not strong enough to permit the prosecutor to mention the defendant's failure to testify in rebuttal. CYes, because the prosecutor is entitled to comment on the state of the evidence. DYes, because even if it was error to comment on the defendant's failure to testify, the error was harmless beyond a reasonable doubt.

The defendant's conviction will likely not be upheld because the prosecutor's comment improperly burdened the defendant's assertion of his privilege against self-incrimination. The prosecution is not allowed to comment on the defendant's failure to testify at trial, because the defendant is privileged under the Fifth Amendment to remain silent. (B) is incorrect because no amount of attacks on the credibility of prosecution witnesses will justify such a comment as a rebuttal. (C) is incorrect because the Fifth Amendment privilege outweighs the prosecutor's right to comment on the state of the evidence. (D) is not the best answer even though the harmless error test does apply to improper comments by the prosecution (i.e., the conviction will not be overturned if the prosecution can show beyond a reasonable doubt that the comments did not affect the outcome of the case). Because there is no real indication as to the strength of the case against the defendant, it is impossible to conclude that the error was harmless beyond a reasonable doubt.

A defendant was arrested on suspicion of running an illegal "moonshine" operation. After taking the defendant back to the police station, an officer began questioning the suspect, thinking that his partner had already given the defendant a Miranda warning. The defendant voluntarily confessed to each and every element of the crime. At trial, the defendant took the witness stand and testified on his own behalf, declaring that he was innocent and that a distillery that the officers found at his home belonged to someone else. The prosecution, on cross-examination, produced the confession that the defendant gave concerning his illegal activities. The defense counsel objected to the admission of the confession. How should the court rule on the defendant's objection? response - correct ASustained, because all evidence obtained in violation of Miranda rights is inadmissible. BSustained, because the prosecution did not get permission from the court in advance to use the confession for any purpose. COverruled, because the prosecution may question the defendant on cross-examination concerning any issue that was brought out in his defense. Correct DOverruled, but the confession should be admitted only for the limited purpose of impeachment.

The defendant's objection should be overruled. A confession obtained in violation of Miranda, but otherwise voluntary, can be used for the limited purpose of impeaching a defendant who testifies at trial. In contrast, an involuntary confession cannot be used to impeach. Here, there are no facts to indicate that the defendant's statement was involuntary. Thus, it can be used to impeach the defendant. (A) is too broad a statement. Although a confession obtained in violation of Miranda is inadmissible in the state's case-in-chief as evidence of guilt, as discussed above, such evidence is admissible for limited purposes. (B) is wrong. Advance permission from the court is not a requirement if the confession is used to impeach. (C) is a correct statement but it does not speak directly to the issue of whether the confession is admissible and to what extent.

In most states, a defendant may be convicted of the principal offense and _______. response - correct A A solicitation to commit that offense Correct B A conspiracy to commit that offense C An attempt of that offense DA conspiracy, attempt, or solicitation to commit that offense

The doctrine of merger has been abandoned in many jurisdictions in cases involving a conspiracy, allowing an accused to be convicted of both conspiracy and the principal offense. However, an accused cannot be convicted of either attempt or solicitation and the principal offense.

Acting on an anonymous telephone call, police went to the defendant's apartment, knocked on the door, and demanded to search it for narcotics. When the defendant refused, the police forced the door open and placed him under arrest. As they were removing him from the apartment, the defendant offered to give the officers "valuable information" in exchange for his release. Before he could say anything else, the defendant was given Miranda warnings by the police. Thereafter, he told the police that he had stored some heroin in his friend's apartment and that he and his friend had been going to sell it. The heroin was recovered, and the defendant was prosecuted for conspiracy to sell narcotics and for possession of narcotics. At his trial, the defendant moved to suppress his statements. Which of the following is the defendant's best argument in support of the motion to suppress? response - correct AThe defendant is entitled to know the identity of his accuser, and the state cannot supply this information. BThe police should have given the defendant Miranda warnings prior to entry into the apartment, and the warnings were ineffectual once the defendant offered to give the police information. CThe defendant was intimidated by the forced entry into the apartment, and because the statements were involuntary and coerced, their use against him would violate due process of law. Correct DThe statements were fruits of an unlawful arrest, and though the Miranda warnings may have been sufficient to protect his right against self-incrimination, they were not sufficient to purge the taint of the illegal arrest.

The entry into the defendant's apartment and his arrest, without a warrant, probable cause, or circumstances permitting an exception from these requirements, were unconstitutional. The statements he made thereafter were fruits of the original unconstitutional arrest and must be suppressed unless the taint was purged. The giving of Miranda warnings was not sufficient. Hence, (D) is the best answer. If probable cause for a warrant is based on information from an informer, usually that informer's identity need not be revealed. Thus, (A) is incorrect. (B) is a misstatement of law. There was no interrogation by the police to trigger the Miranda requirements. (C) is attractive but not as accurate an answer as (D). If the police had been acting with probable cause to arrest, their forced entry into the apartment would not have made the defendant's statements involuntary.

The police, suspecting that the defendant was dealing drugs, observed several people walk up to the defendant's door, knock on his door, and then exchange cash for small packages that the police believed contained drugs. Two uniformed police officers then walked up to the door and knocked. The defendant answered the door, and one police officer asked if they could come in and take a look around. The defendant, believing that he had no other choice but to let the officers inside, agreed. Once inside, they discovered equipment used for making methamphetamine and several tablets of methamphetamine that were sitting on a table covered by a bed sheet. One officer promptly arrested and handcuffed the defendant while the other seized the equipment and tablets. Prior to his trial for the illegal manufacture and possession of methamphetamine, the defendant moved to suppress the evidence as having been illegally seized. Should the motion be granted? response - correct Correct ANo, because the defendant allowed the police officers to enter his home and look around. BNo, because exigent circumstances existed for the warrantless seizure of the evidence. CYes, because the police should have secured the area and obtained a warrant to seize the evidence. DYes, because the defendant's consent was not voluntary.

The evidence should not be suppressed because the defendant consented. To be reasonable under the Fourth Amendment, most searches must be pursuant to a warrant. The warrant requirement serves as a check against unfettered police discretion by requiring the police to apply to a neutral magistrate for permission to conduct a search. A search conducted without a warrant will be invalid (and the evidence discovered during the search generally must be excluded from evidence) unless the search and seizure falls within an exception to the warrant requirement. One exception to the warrant requirement is when the police have valid consent to search the premises. The police may conduct a valid warrantless search when they have a voluntary consent to do so. Knowledge of the right to withhold consent, while a factor to be considered, is not a prerequisite to establishing a voluntary consent. In the instant case, there are no facts that indicate that the police put any undue pressure on the defendant to consent to the search. Although it is a factor to be considered in determining whether the consent was voluntary, the defendant's subjective mistake about being able to withhold consent would probably not, by itself, be sufficient to deem the consent involuntary. As a result, (A) is the correct answer, and (D) is incorrect. (B) is incorrect. The Supreme Court has made it clear that there is no general "emergency" exception to the warrant requirement, although the police may seize "evanescent" evidence in certain circumstances. That said, there is no indication that evidence here would disappear, as it seems that the defendant's operation was ongoing, thus giving the police time to get a warrant. (C) is incorrect because the police may conduct a warrantless search with the defendant's permission, as they did in this question.

A farmer was in the middle of plowing his field when his tractor broke down. While attempting to repair it, he discovered that he needed a special wrench. He knew that his neighbor used the same type of tractor and kept a large cache of tools in his basement. Not wanting to make the long drive into town to buy one wrench that he probably would not use much, the farmer went to his neighbor's house to borrow the wrench. However, no one was home so he decided to look in his neighbor's basement for the wrench, thinking that he would return it before the neighbor came back. To gain entry, the farmer opened an unlocked window and climbed through the opening to the basement. Once inside, the farmer found the tool and took it with him to work on the tractor. His neighbor returned soon after and contacted the police when he discovered that one of his tools was missing. The police determined that the farmer took the tool and he was charged with burglary. What is the farmer's best defense against that charge in a common law jurisdiction? response - correct ANobody actually lived in the basement. BThe farmer knew that the house was unoccupied and would not have entered without permission had the neighbor been home. CThe farmer entered the house through an unlocked window. Correct DThe farmer intended only to keep the wrench for a couple of hours.

The farmer's best defense is that he intended only to keep the wrench for a couple of hours. Given that the farmer intended merely to borrow the tool, he lacked the intent to commit larceny, and thus would not be guilty of burglary. Common law burglary consists of: (i) a breaking; (ii) and entry; (iii) of the dwelling; (iv) of another; (v) at nighttime; (vi) with the intent of committing a felony therein. The farmer entered his neighbor's house intending to remove the tool. Thus, the facts indicate that the only felony he could have intended to commit at the time of entry would be larceny. Larceny consists of: (i) a taking; (ii) and carrying away; (iii) of tangible personal property; (iv) of another; (v) by trespass; (vi) with intent to permanently (or for an unreasonable time) deprive the person of his interest in the property. At common law, if the defendant intended to return the property within a reasonable time, and at the time of the taking had a substantial ability to do so, such an unauthorized borrowing would not constitute larceny. Consequently, if the farmer intended to keep the tool only for the short time to fix his tractor, then he did not intend to permanently deprive his neighbor of his interest in the wrench. Because the farmer thus lacked the intent to commit a felony in his neighbor's home at the time he entered, the farmer would not be guilty of burglary. (A) is incorrect because, for purposes of the crime of burglary, a structure is deemed to be a dwelling simply if any part of it is used regularly for sleeping purposes. Thus, the fact that nobody lived in the basement is irrelevant. (B) is incorrect because the fact that the house was unoccupied is irrelevant to his culpability for burglary. The crime of burglary would have been complete if the farmer had broken and entered his neighbor's home with the intent of committing a felony therein, regardless of whether the home was currently unoccupied. Consequently, the farmer's knowledge that the house was unoccupied would provide him with no defense to a charge of burglary. (C) is incorrect because the breaking needed for burglary requires only minimal force to gain entry. Opening an unlocked window is a sufficient use of force to constitute a breaking.

Two friends entered a bar looking to get money to pay off a loan shark, but with no plan how to do so. They struck up conversations with two women. The first friend left the bar, having induced one of the women to return home with him. Once in his house, the first friend told the woman that she would not be allowed to leave unless she gave him all of her money. Fearing for her safety, the woman gave him all of the cash she had in her possession. Meanwhile, the second friend remaining at the bar noticed that the other woman left her credit card on the counter. When the woman looked away, the friend picked up the credit card and put it into his pocket. Shortly thereafter, the woman realized her card was gone and accused the man of taking it. The man pretended to be insulted, slapped the victim, and went off with the credit card in his pocket. Which of the two friends can be convicted for common law robbery? response - correct ABoth can be convicted. Correct BThe first friend can be convicted, but the second cannot be convicted. CThe second friend can be convicted, but the first cannot be convicted. DNeither of the two friends can be convicted of robbery.

The first friend can be convicted of common law robbery, but the second cannot be convicted. Robbery is the taking and carrying away of the personal property of another from the other's person or presence by force or intimidation. In the instant case, the first friend committed a robbery when he threatened the woman and told her that she could not leave without giving him all of her money. The only issue would be whether the asportation element is satisfied. However, the asportation element is satisfied by any slight moving, and it is likely that the first friend moved the money at some point during the robbery. Thus, a jury could find the first friend guilty of robbery. This makes (C) and (D) wrong answer choices. In the case of the second friend, however, a conviction for robbery is unlikely. Although a close call, the taking or retention of the property was not by force or intimidation in the second case. The crime against the property was already completed when the man slapped the victim. Furthermore, the slap was not to prevent the woman from physically taking the credit card back; rather, it was a ruse used to deflect the accusation that the second friend took the credit card. Thus, (A) is wrong, and this provides a second reason why (C) is incorrect.

In a property settlement after a divorce, the wife was awarded all personal property that had been accumulated during the marriage, including the husband's classic 19-inch black-and-white TV set. In order to get his prized TV set back, the husband lied to his friend, telling him that the wife took the TV set in violation of the property settlement. The friend remembered that the wife gave the friend's wife a key to her new home, and he volunteered to go with the husband to get the TV back while the wife was at work. The husband and the friend went to the wife's house, but, unbeknownst to them, the wife had taken the day off work. After the friend noisily opened the back door with his wife's key, the wife called the police, who quickly arrived and arrested the husband and the friend. As to a charge of common law conspiracy to commit larceny, how should the friend be found? response - correct Correct ANot guilty, because he did not intend to steal. BNot guilty, because he did not have a corrupt motive. CGuilty, because there was an agreement, and the opening of the locked door was sufficient for the overt act. DGuilty, because good motives are not a defense to criminal liability.

The friend should be found not guilty because he did not intend to steal. At common law, conspiracy consists of (i) an agreement between two or more persons, (ii) an intent to enter into an agreement, and (iii) an intent to achieve the objective of the agreement. The object of the agreement must be something unlawful. Here, the friend did not intend to achieve the objective of the conspiracy-to permanently deprive the owner of her property-because the friend thought the husband was the true owner of the TV. (C) is incorrect because there must be an agreement to reach an unlawful objective. Because the friend thought he was achieving a lawful objective, he did not have the intent required for conspiracy. (B) is incorrect because a "corrupt motive" is not an element of a crime. A person could be found guilty of a crime even if he did not have a corrupt motive, assuming all required elements for a crime are present. (D) is incorrect for a similar reason. "Good motive" is largely irrelevant; the intent, or lack thereof, is what is important.

The owner of a garage and one of his mechanics had a dispute over the amount of wages due for work that the mechanic had done on cars. The owner alleged that he paid the mechanic for work that the mechanic did not perform, while the mechanic contended that she did the work. After arguing over the matter for over an hour, the mechanic quit. The owner did not allow the mechanic to take her tools with her, stating that he was keeping her tools until she repaid the money that he had overpaid her. The mechanic met with a friend and discussed the situation with him. After hearing what had happened, the friend, believing that the owner was unlawfully retaining the mechanic's tools, suggested that he could pose as a garage client and retrieve the tools for the mechanic, and the mechanic agreed. The plan succeeded, except for the fact that the friend took tools that belonged to a new employee. The owner immediately realized what had happened, and he gave the friend's license number to the police. The friend was arrested before he could show the tools to the mechanic. Of the following, which is the best argument for the friend as a defense to a charge of larceny in a common law jurisdiction? response - correct AHe had the consent of the tools' owner. Correct BHe thought the tools belonged to the mechanic. CHe intended to return the tools to the mechanic rather than keep them. DHe was apprehended by the police before the mechanic could inspect the tools.

The friend's best argument is that he thought the tools belonged to the mechanic. Larceny is the taking and carrying away of the tangible personal property of another by trespass with the intent to permanently (or for an unreasonable time) deprive the owner (or person in possession) of his interest in the property. Thus, larceny is a specific intent crime, in that the defendant must intend to permanently deprive the person of his interest in the property. Importantly, this intent generally must exist at the time of taking (save for the continuing trespass doctrine). In this case, the friend believed that the mechanic was being unlawfully deprived of his property, and his intent was to return the tools to their rightful owner. Thus, when he took the tools, he lacked the intent to permanently deprive another of his interest in the property. This would be true even if the garage owner factually had a lawful right to retain the tools, as the mistake of law would negate the friend's intent to permanently deprive another of his interest in the property. (A) is wrong, of course, because he did not have the consent of the owner of the tools he took. (C) is wrong because it is not a defense to a charge of larceny that the defendant did not intend to keep the item taken for his own use. (D) is wrong because it prematurely raises the continuing trespass doctrine. Under the continuing trespass doctrine, a defendant may be convicted of larceny when he initially takes property with a wrongful state of mind, but without the intent to permanently deprive the owner of the property, and then later forms the intent to permanently deprive the owner of the property. Here, the friend had not yet had the opportunity to form that later intent (even assuming the initial taking was with a wrongful state of mind). Thus, (D) does not address the correct issue.

A motorist drove home from work late one night, and fell asleep behind the wheel of his car. His car drifted across the middle of the road and struck another car. The other driver was killed instantly in the collision. Angered by the noise of the collision, a homeowner fired a gun out the window of his house at the car. The bullet struck and killed a bystander. Both the motorist and the homeowner were arrested and charged with common law murder. Which of the defendants likely would be found guilty? response - correct ABoth the motorist and the homeowner. BThe motorist. CNeither the motorist nor the homeowner. Correct DThe homeowner.

The homeowner likely would be found guilty. At common law, murder was the unlawful killing of a human being with malice aforethought. Malice aforethought could be established with any one of the following states of mind: intent to kill; intent to cause serious bodily harm; the depraved heart killing (a reckless indifference to an unjustifiably high risk to human life); or the commission of a felony. The homeowner would be guilty of murder. Firing a gun out of his window at a car would demonstrate a reckless indifference to a high risk to human life. Thus, (B) and (C) are incorrect. It is unlikely that the motorist would be guilty of murder. While his action might be classified as negligent or even reckless, it would not represent a depraved heart (reckless indifference to life) state of mind. Thus, (A) and (B) are incorrect.

A wife suffered from a particularly virulent form of cancer, and had lapsed into a nearly comatose state. Because the doctors had indicated that any treatment they could prescribe would be of little value, her husband decided to administer various poisons to his wife, thinking that they might stimulate her natural body defenses, or kill the cancer cells, resulting in her recovery. He tried doses of many different types of poison. Despite his ministrations, his wife died three days later. An autopsy performed by the county coroner established the cause of death as cancer. If the husband is prosecuted for the murder of his wife, which of these is the best reason why he would be acquitted? response - correct AHe was trying to save her life. BHe did not have the necessary malice for his actions to constitute murder. CMedical science had given her up for dead. Correct DHe did not cause her death.

The husband cannot be convicted because he did not cause his wife's death. Murder is defined as the unlawful killing of another human being with malice aforethought. To be guilty of murder, the defendant's action must be both the cause in fact and the proximate cause of the victim's death. The defendant's act will be a cause in fact of death if, but for the defendant's action, the victim would not have died as and when she did. Here, the victim would have died when she did even if the husband had not administered the poison, because she died not from the poison, but only from her cancer. Thus, the husband's actions were not the cause in fact of death, and (D) is correct. (A) and (B) are incorrect because if the other elements of murder are established, administering poison might be sufficient to establish malice aforethought. Malice aforethought for murder can be established by conduct done with the awareness of an unjustifiably high risk to human life, and the husband knew that the poisons were dangerous and could kill. (C) is incorrect because the law forbids shortening a life even for one second, so it is not a defense that medical science had given the victim up for dead. If the defendant's action in any way shortened the victim's life, he can be held liable for murder.

A defendant going to trial for the felony of aggravated battery filed a motion to have the court seat a jury with only five members. He stated that five was his lucky number and that he would be willing to sign any release to ensure a jury of five. Should the judge grant the defendant's motion to impanel a jury of five members? response - correct AYes, the number of jurors is up to the defendant. BYes, the number of jurors is typically six, but five is permitted as long as the verdict is unanimous. Correct CNo, the Constitution requires at least six jurors. DNo, the number of jurors is up to the judge, and the defendant does not have any say in the matter.

The judge should not grant the defendant's motion. The Constitution requires at least six jurors. There is no constitutional right to a jury of 12, but there must be at least six jurors to satisfy the right to jury trial under the Sixth and Fourteenth Amendments. [Ballew v. Georgia (1978)] (A) is incorrect because, even were a defendant to waive his right to a jury of six, the judge could not constitutionally impanel such a jury. (B) is incorrect because, as stated above, at least six jurors are required by the Constitution. Note also that six-person juries must be unanimous, although 12-person juries need not be. [Burch v. Louisiana (1979)] (D) is incorrect because the number of jurors is not up to the judge in the sense that a judge may not waive the six-juror minimum. In most jurisdictions, the number of jurors (above six) is established by statute or court rule, not by the judge.

The police received information linking a man to drug trafficking and went to the man's residence, where he lived with his mother. The police found the mother at home, and she told them that her son was not expected back until later. The police informed the mother that they suspected the man of selling drugs and asked if they could search his room. She replied, "I'm finished with that no-good bum; not only is he into drugs, but he has been stealing my money to pay for them, and all the time I'm making his bed and fixing his food. You can search his room. He likes to keep his private stuff under his pillow. I hope he goes to jail." The police searched the man's room and discovered a quantity of marijuana under the pillow of his bed. If before trial the man's attorney moves to suppress the marijuana on grounds that the search was invalid, should the court grant the motion? response - correct AYes, because the man had a legitimate expectation of privacy in the area searched, and the police did not have a warrant. BYes, because the man's mother's consent was given at a time when police knew her interests were in conflict with the man's. Correct CNo, because the man's mother had the authority to consent to the search of his room. DNo, because with the mother's statement the police had probable cause to search the room.

The man's motion to suppress should be denied because his mother had authority to consent to the search of his room. A search of a residence can be based on the voluntary consent of the occupant. Where a parent has general access to a room occupied by a son or daughter, the parent can give a valid consent to a general search of the room even if the son or daughter is an adult. The facts in the question indicate that the man's mother had general access to his room ("and all the time I'm making his bed"). Therefore, her consent is valid and eliminates the need for probable cause and a warrant. (A) is wrong. The man had a legitimate expectation of privacy in the area searched, but the consent of his mother eliminated the need for a warrant. (B) is wrong. At one time, some courts required an "amicable relationship" between the parties before the police could rely on a third party's consent. The "amicable relationship requirement" is no longer recognized by the courts. (D) is not a good answer. It is true that with the mother's statement the police had probable cause to search the man's room. However, probable cause alone would not validate the search. The police would need probable cause plus a warrant or a valid consent. In this question the search would have to be based on consent.

A defendant held up a gasoline station. During the robbery, he shot and killed a customer who attempted to apprehend him. The defendant was prosecuted for premeditated murder and convicted. Thereafter, he was indicted for armed robbery of the station. Before the trial, his attorney moved to dismiss the indictment on the ground that further proceedings were unconstitutional because of the defendant's prior conviction. Should the motion to dismiss be granted? response - correct AYes, because once the defendant was convicted on any of the charges arising out of the robbery, the prosecution was constitutionally estopped from proceeding against the defendant on any charge stemming from the same transaction. BYes, because the Double Jeopardy Clause prohibits a subsequent trial on what is essentially a lesser included offense. Correct CNo, because there is no constitutional requirement that all known charges against a defendant be brought in the same prosecution. DNo, because estoppel does not apply when a defendant is charged with violating two different statutes.

The motion to dismiss should be denied. For purposes of the Double Jeopardy Clause, two crimes do not constitute the "same offense" if each crime requires proof of an additional element that the other crime does not require, even though some of the same facts may be necessary to prove both crimes. Here, even though the same facts are involved for both crimes, the robbery charge requires proof of a taking by force but not a death, while the murder charge requires proof of a death but not of a taking of property. Thus, (C) is correct and (A) is incorrect. (B) is incorrect because armed robbery is not a lesser included offense of premeditated murder. (D) is incorrect because the prosecution would be estopped if violation of one statute constituted a lesser included offense of the other statute.

The president of a private college received a report that there was a great deal of cocaine use occurring on the second floor of the dormitory. The president persuaded the school security officers to place several concealed microphones in the second-floor student lounge. Conversations occurring in the lounge were monitored by the security officers and they recorded a conversation in which a sophomore at the college offered to sell cocaine to a freshman. A tape of the conversation was taken to the local police, who played it for a local judge. The judge issued a warrant to search the sophomore's room. While searching the room the police discovered a large amount of cocaine and the sophomore was arrested and charged with unlawful possession of narcotics. His attorney moved to prevent the introduction of the cocaine into evidence. Will the motion most likely be granted? response - correct AYes, because the sophomore's privacy was unreasonably invaded. BYes, because the electronic surveillance was "fundamentally unfair." Correct CNo, because the police properly obtained a search warrant. DNo, because the college president was acting on behalf of the college population in general.

The motion to suppress should be denied because a valid search warrant was obtained. A search warrant must be based on probable cause. Here, there was sufficient information for a judge to conclude that there was probable cause to believe that evidence of a crime would be found in the sophomore's room. Thus, the warrant was properly obtained. (A) is wrong because the sophomore had only a limited expectation of privacy in a dormitory's lounge. Also, even if his privacy had been invaded, any invasion here was done by private persons, not the state, and thus would not prevent introduction of the evidence. (B) is wrong because it is untrue as a matter of law. (D) is wrong because it is not a sufficient basis to deny the sophomore's motion. It is irrelevant that the college president acted on behalf of the college population.

The Fourth Amendment prohibits unreasonable searches and seizures. Generally, for a search to be valid, it must be pursuant to a warrant issued by a neutral and detached magistrate and based on probable cause to believe that seizable evidence or fruits of a crime will be found on the premises to be searched. However, the Supreme Court has carved out a number of exceptions to this rule. Under the exception to the warrant requirement for searches incident to arrest, which of the following statements is true? response - correct A The exception applies only if the police fear for their safety. Correct B The exception applies after any constitutional arrest. C If the arrest violates state law, the exception does not apply. D The exception applies only in cases of arrest for felonies.

The police may conduct a search incident to arrest after any constitutional arrest. A warrantless search can be conducted following an arrest that violates state law. The police may conduct a search incident to an arrest whenever they arrest a person, even if the arrest is invalid under state law, as long as the arrest was constitutionally valid (e.g., reasonable and based on probable cause). It is not necessary that the police fear for their safety for a warrantless search to be conducted following an arrest. The police need not actually fear for their safety or believe that they will find evidence of a crime as long as the suspect is placed under arrest. It is not true that the exception applies only in the case of felony arrests. As discussed above, the exception applies after any lawful arrest.

A 12-year-old girl entered a grocery store. When she believed that no one was looking, she grabbed two candy bars and concealed them under her coat. As she attempted to leave the store, a security guard employed by the store grabbed the girl by the arm. He told her, "You're too young to be a thief!" The girl began crying and blurted out, "I lost my lunch money on the way to school and I was really hungry!" If the girl is charged with shoplifting, what is the prosecution's best argument that her Miranda rights have not been violated by the security guard? response - correct AThe guard's statement was not interrogatory. BIt is discretionary whether to give juveniles Miranda warnings. CThe girl has not yet been arrested and formally charged with any crime. Correct DThe security guard was not a government agent.

The prosecution's best argument is that, because the security guard was not a government agent, he need not have given the girl Miranda warnings. As a means of protecting the Fifth Amendment privilege against compelled self-incrimination, a person must be informed prior to custodial interrogation that: (i) she has the right to remain silent; (ii) anything she says can be used against her in court; (iii) she has the right to the presence of an attorney; and (iv) if she cannot afford an attorney, one will be appointed for her if she so desires. These Miranda warnings must be given only if the detainee is being questioned by someone known to be working for the police. Here, the security guard is employed by a private business (the grocery store). Thus, since the guard was not required to inform the girl of the Miranda warnings, he could not possibly have violated the girl's Miranda rights. (A), (B), and (C) all reflect matters that would come into play only if the guard were a government agent. However, they would also be incorrect even if the guard were a government agent. (A) is incorrect because the guard's statement might be deemed to be interrogatory. "Interrogation" refers not only to express questioning, but also to any words or actions on the part of the police that they should know are reasonably likely to elicit an incriminating response from the suspect. Although it is not an express question to say "You're too young to be a thief," such words are reasonably likely to bring forth some sort of incriminating response. Therefore, it is incorrect to state that the guard's statement was not interrogatory. (B) is incorrect because, in a delinquency proceeding, a juvenile must be afforded the right not to testify, including all aspects of the privilege against self-incrimination. Miranda warnings are a very important aspect of the privilege against self-incrimination. In addition, a juvenile court may determine that a child should be transferred to adult court for trial as an adult on criminal charges. Certainly, in such an instance, the requirement of Miranda warnings would be applicable. Consequently, regardless of whether a child is treated by the courts as a juvenile or an adult, it is incorrect to state that giving the Miranda warnings to juveniles is discretionary. (C) is incorrect because an actual arrest and formal charges are not a prerequisite to triggering the need for Miranda warnings. All that is required is that an interrogation be custodial in nature. An interrogation may be considered custodial if a reasonable person under the circumstances would feel that she is not free to terminate the interrogation and leave. The security guard grabbed the girl by the arm before making his statement. Thus, it could be argued that what followed was custodial interrogation, because the girl did not believe she was free to leave.

A man was driving very erratically when he was stopped by state troopers and arrested for drunk driving. He was advised of his constitutional rights and elected to remain silent. At trial for his drunk driving charge, the man testified in his own defense, stating that he had just left his doctor's office and had been administered medication without being told that it would seriously and immediately hamper his coordination. On cross-examination, the prosecutor asked whether the defendant just made up this medication story after the fact to evade legitimate liability for driving while intoxicated and the man said he had not. The prosecutor then asked why the defendant had not told the arresting officer about the medication, and defense counsel objects. The trial court should rule that the question is: response - correct AImproper, because to require the defense to inform the prosecution of defendant's testimony prior to trial would be unconstitutional pretrial discovery. Correct BImproper, because use of defendant's post-arrest silence violates his right to due process of law. CProper, because defendant's silence was not used as direct evidence but only for impeachment on cross-examination. DProper, because defendant's post-arrest silence is a prior inconsistent statement which is admissible to show recent fabrication.

The question was improper because it effectively comments on the defendant's post-arrest silence and thus violates his right to due process of law. A prosecutor may not comment on the defendant's silence after being arrested and receiving Miranda warnings. The warnings carry an implicit assurance that silence will carry no penalty. Thus, the defendant's invocation of his right to remain silent cannot be used as evidence against him at trial, nor can he be questioned or cross-examined about his decision to remain silent, even if he testifies at trial. Thus, (B) is the correct answer, and (C) is wrong. (Distinguish: If the defendant fails to disclose potential exculpatory evidence to the police after waiving his right to remain silent, he may be cross-examined at trial on that failure in an effort to show that it is a recent fabrication. In this case, however, the defendant invoked his right to remain silent.) (A) is wrong; the question is improper, but not because it would be unconstitutional pretrial discovery. (D) is also wrong. Although statements made without proper Miranda warnings can sometimes be used to impeach, the defendant's silence would not be classified as a prior statement.

The right to be free of double jeopardy for the same offense arises from __________. response - correct A the Fourth Amendment Correct B the Fifth Amendment C the First Amendment D the Sixth Amendment

The right to be free of double jeopardy derives from the Fifth Amendment and has been incorporated into the Fourteenth Amendment. Under this right, once jeopardy attaches, the defendant may not be retried for the same offense. The right to be free of double jeopardy does not arise from the First Amendment, the Fourth Amendment, or the Sixth Amendment. It arises from the Fifth Amendment.

A car was pulled over for speeding and the police officer ordered the driver to step out and move away from the car. The officer subsequently searched the glove compartment of the car and found an illegal handgun. Which of the following would NOT be a valid basis for the search of the glove compartment? response - incorrect AThe officer had a reasonable suspicion that the driver was dangerous. Correct BThe officer properly placed the driver under arrest and secured him in the back of the squad car before conducting the search. Incorrect CThe officer properly placed the driver under arrest, impounded the vehicle, and then conducted a standard inventory search. DThe officer had probable cause to believe that the vehicle contained illegal narcotics.

The search of the glove compartment would not be valid if the officer had placed the driver under arrest and secured him in the back of the squad car. After arresting the occupant of a vehicle, the police may search the interior of the vehicle incident to the arrest if (i) the arrestee is unsecured and still may gain access to the interior of the vehicle, or (ii) the police reasonably believe that evidence of the offense for which the person was arrested may be found in the vehicle. Here, if the officer secured the arrested driver in the back of the squad car, the first basis for permitting the search incident to arrest no longer applies (and there are no facts to support the second basis). Hence, there would be no grounds under (B) to support the search of the glove compartment. (A) is incorrect because, under the stop and frisk exception, an officer may order the occupant of a vehicle out of the car and search the passenger compartment if the officer reasonably believes that the occupant may be dangerous. (C) is incorrect because once a vehicle has been impounded, the police may conduct an inventory search as part of standard procedure. (D) is incorrect because, under the automobile exception, the police may search a vehicle without a warrant if they have probable cause to do so, and they may seize any contraband that they find.

A student and a few of his friends were making their way to spring break. Along the way, the old van that they were driving broke down. Not wanting to miss any part of spring break festivities, the student asked the mechanic on duty at the repair shop for a rush job. The mechanic provided the student with a repair estimate, and the student, on the basis of the estimate, authorized the repair and promised to pay when he came back to pick up the van. When the mechanic called the student to tell him that the van was repaired, the student, rather than paying for the repair, told one of his friends that the mechanic had agreed to finance the repair charges and that the only thing left to do was pick up the van in the garage's parking lot. The student handed the friend a key to the van and told him to go pick the van up so that they could continue their trip to spring break. The friend did so. The mechanic makes a criminal complaint against the student for larceny of the van. If the case is prosecuted, will the student likely be found guilty? response - correct ANo, because it was the student's van to begin with. BNo, because the friend took the van. Correct CYes, because the friend took the van from the mechanic without the mechanic's knowledge or permission. DYes, because the student promised to pay the mechanic for his work when he came to get the van.

The student will most likely be found guilty. Larceny is the taking and carrying away of the personal property of "another" with the intent to permanently deprive the other person of the property. It is possible to commit larceny of your own property if another person, such as a bailee, has a superior right to possession of the property at that time. Because the mechanic had a right to possession of the van until he was paid, the student committed larceny when he had his friend take the van without the mechanic's consent. (B) is wrong because a person can be guilty even though he did not personally engage in the behavior if he acts through an innocent agent. (D) is also incorrect. The student is guilty, but not for the reason stated in (D). He would be guilty even if he had not made the promise to pay for the van; he incurred an obligation to pay by having the repairs done.

The Fourth Amendment prohibits unreasonable searches and seizures. The Supreme Court has held that only a person with a reasonable expectation of privacy in a place searched or an item seized can challenge a search or seizure on Fourth Amendment grounds to bar the use of the fruits of the search as evidence at trial. An intrusion into a place or thing in which the defendant does not have a reasonable expectation of privacy is not considered a search for Fourth Amendment purposes. Which of the following is most likely to be found to constitute a search under the Fourth Amendment? response - correct A A search of a garbage can placed on a curb for collection. Correct B Use of a thermal imager to photograph the interior of a home. C A dog sniff of a car during a traffic stop. D Low altitude aerial photographing of a home using a telephoto lens.

The use of a thermal imager to take photographs of the interior of a home most likely constitutes a search. The Supreme Court has held that there is a strong expectation of privacy within one's home, so obtaining information about the interior of a home through sense-enhancing technology that could not otherwise have been obtained without a physical intrusion constitutes a violation of the Fourth Amendment, absent a warrant. In contrast, the Supreme Court has held that one does not have an expectation of privacy in the smell of one's auto. Thus, a dog sniff of a car during a traffic stop does not constitute a search or implicate the Fourth Amendment, as long as the police lawfully stopped the car and the stop does not extend beyond the time necessary to issue a ticket and conduct ordinary inquiries. A warrantless search of a garbage can placed on a curb for collection also does not constitute a violation of the Fourth Amendment. Under the Supreme Court's "open fields" doctrine, areas outside the curtilage (dwelling house and outbuildings) are subject to police entry and search, because they are held to be open to the public and thus unprotected by the Fourth Amendment. Lastly, the Supreme Court has held that one does not have a reasonable expectation of privacy in those areas of one's yard that can be viewed from public airspace, so the police may take aerial photographs of a suspect's house and yard, even at low altitude and with a telephoto lens and not implicate the Fourth Amendment, at least as long as the police are within legal airspace and the lens is in general public use.

The warnings required under Miranda ________ include that the detainee has the right to __________. response - correct A Do not; remain silent Correct B Do not; be informed of the charges against him C Do; confront witnesses against him D Do; an impartial decisionmaker

The warnings required under Miranda do not include that the detainee has a right to be informed of the charges against him. Also, it is not true that the warnings must include that the detainee has the right to an impartial decisionmaker. While such a right exists under the Due Process Clause, Miranda does not require that detainees be informed of that right. Nor is it true that the warnings must include that the detainee has the right to confront witnesses against him. While there is such a right under the Sixth Amendment, Miranda does not require that detainees be informed of that right. The warnings required under Miranda do require that the warnings inform the detainee of the right to remain silent. The full warnings must inform the detainee that: (i) He has the right to remain silent; (ii) Anything he says can be used against him in court; (iii) He has the right to the presence of an attorney; and (iv) If he cannot afford an attorney, one will be appointed for him if he so desires. Note that Miranda warnings do not need to be given verbatim, as long as the substance of the warning is there.

A man and woman agreed to burn down a neighbor's house in retribution for some wrong the neighbor allegedly committed against them. Both the man and woman were arrested shortly after they poured gasoline on the neighbor's front porch. The man revealed to the police that he participated in the plan to ensure that nothing bad would happen to the neighbor, and that he had made an anonymous telephone call to the police alerting them to the crime, which enabled the police to arrest him and the woman "in the act." The woman stated that she would not have participated if not for the man's encouragement. If the woman is charged with a conspiracy at common law to commit arson, how should she be found? response - correct ANot guilty, because she was not predisposed to commit the crime but for the man's encouragement. Correct BNot guilty, because the man did not intend to commit arson. CGuilty, because there was an agreement, and pouring gasoline on the front porch was sufficient for the overt act. DGuilty, because arson is not a specific intent crime.

The woman should be found not guilty of a conspiracy to commit arson. To be convicted of a conspiracy at common law, it must be shown that at least two persons agreed to achieve an unlawful objective. Having two or more persons is a necessary element of conspiracy under the traditional bilateral approach. Here, the facts indicate that the man did not intend to achieve the objective of the conspiracy-to burn the dwelling house of another. Thus, the woman cannot be guilty of conspiracy to commit arson. (C) is incorrect. The man feigned his agreement, making the answer factually inaccurate. (D) is also incorrect. Although it is true that arson is not a specific intent crime, conspiracy is a specific intent crime, in that the prosecution must show that the defendant intended to agree and intended to achieve the unlawful objective. Thus, the fact that the underlying crime is not a specific intent crime is irrelevant. (A) is incorrect. Even if the woman would not have committed the crime without the man's inducement, that is not a defense for the woman. A person cannot be entrapped by a private citizen.

A woman was arrested outside of a house shortly after she had broken in and stolen some jewelry. She was indicted for larceny and later for burglary. She was tried on the larceny indictment and convicted. Thereafter, she was brought to trial on the burglary indictment. Relying on the Double Jeopardy Clause of the Constitution, the woman moves to dismiss the indictment. Should the motion be granted? response - correct AYes, because the Double Jeopardy Clause requires that all offenses arising out of the same transaction be adjudicated in the same trial. BYes, because the Double Jeopardy Clause allows the imposition of separate sentences for separate offenses occurring during the same criminal episode only if the offenses are tried together. Correct CNo, because larceny and burglary are offenses that may constitutionally be tried and punished separately, even if they arise out of the same transaction, because each requires proof of a fact that the other does not. DNo, because the only protection double jeopardy affords to a defendant charged with multiple counts is under the doctrine of collateral estoppel.

The woman's motion should be denied because the Double Jeopardy Clause does not prohibit the second prosecution. The Double Jeopardy Clause of the Fifth Amendment provides criminal defendants with the right to be free of double jeopardy for the same offense. However, two crimes do not constitute the same offense if each crime requires proof of an additional element that the other crime does not require, even though some of the same facts may be necessary to prove both crimes. [Blockburger v. United States (1932)] Here, larceny requires a taking and carrying away of the property of another, which burglary does not require, and burglary requires a breaking and entry, which larceny does not require. Hence, they are distinct offenses for purposes of the Double Jeopardy Clause. (A) and (B) are incorrect because the Supreme Court does not use a "same transaction" or "same episode" test suggested by these answer choices; instead, the Blockburger test is used regardless of whether the two offenses were tried together at a single trial or at separate trials. (D) is incorrect because while double jeopardy also protects against inconsistent factual determinations at a subsequent trial, it protects against multiple prosecutions as well, as long as the crime is the "same offense."

The following are all exceptions to the Fourth Amendment warrant requirement, allowing for warrantless searches in situations where quick action is needed: response - correct A Evanescent evidence; emergencies that threaten health or safety; investigation of a murder scene B Hot pursuit of a fleeing felon; emergencies that threaten health or safety; investigation of a murder scene Correct C Hot pursuit of a fleeing felon; evanescent evidence; emergencies that threaten health or safety D Hot pursuit of a fleeing felon; evanescent evidence; investigation of a murder scene

There is no exception to the warrant requirement based on the need to investigate a murder scene. Generally, if the owner of the location where the body was found objects, a warrant must be obtained. There is an exception to the warrant requirement for hot pursuit of a fleeing felon. Police officers in hot pursuit of a fleeing felon may make a warrantless search and seizure. They may even pursue the suspect into private dwellings. There also is an exception to the warrant requirement for evanescent evidence, such as scrapings of tissues from under a suspect's fingernails, which could be washed away. Whether such a warrantless search is reasonable is judged by the totality of the circumstances. If by its nature the evidence is likely to disappear before a warrant may be obtained, the evanescent evidence exception applies. Finally, there is an exception to the warrant requirement when the police are acting in emergency situations threatening immediate health or safety. This includes situations where the police see someone injured or threatened with injury. For example, if a police officer is answering a public disturbance call and sees someone being beaten within the premises when the officer arrives, the officer can go in without a warrant to help the victim of the beating. This is often referred to as the community caretaker exception.

In a criminal trial, what is the minimum number of jurors allowed under the Sixth and Fourteenth Amendments? response - correct A Nine Correct B Six C Twelve D Seven

There must be at least six jurors to satisfy the right to a jury trial under the Sixth and Fourteenth Amendments.

Felony murder generally requires that: response - correct A The killing be committed during the course of the felony, the death must have been a foreseeable result of the felony, and the defendant must be convicted of the underlying felony Correct B The killing be committed during the course of the felony, the felony must be independent of the killing, and the death must have been a foreseeable result of the felony C The killing be committed during the course of the felony, the felony must be independent of the killing, and the defendant must be convicted of the underlying felony D The death must have been a foreseeable result of the felony, the felony must be independent of the killing, and the defendant must have been convicted of the underlying felony

To convict a defendant of felony murder, the prosecution must prove, beyond a reasonable doubt, that the defendant committed a felony (i.e., he is factually guilty of the felony). However, the defendant need not actually be convicted of the underlying felony if the statute of limitations for the felony has expired. The killing must take place while the felony is being committed. When the defendant reaches "a place of temporary safety," the felony is deemed terminated. The felony must be independent of the killing (e.g., the felony of manslaughter cannot be the underlying felony for felony murder). Finally, most states require that the death must be a foreseeable result of the commission of the felony.

Under common law, the elements of a conspiracy include: response - correct A An agreement between two or more persons, an intent to enter into an agreement, an intent to achieve the objective of the agreement, and completion of the crime originally agreed upon. Correct B An agreement between two or more persons, an intent to enter into an agreement, and an intent to achieve the objective of the agreement. C A written agreement between two or more persons, an intent to enter into an agreement, and an intent to achieve the objective of the agreement. D An agreement between two or more persons, an intent to achieve the objective of the agreement, and completion of the crime originally agreed upon.

Under common law, the elements of conspiracy include an agreement between two or more persons, an intent to enter into an agreement, and an intent to achieve the objective of the agreement. The agreement between the parties need not be written. Although most states require some overt act in furtherance of the conspiracy for the crime of conspiracy to be complete, the crime agreed upon does not need to be completed for the conspirators to be guilty of conspiracy.

Which of the following statements is true with regard to the execution of a search warrant? response - correct A A third party may never accompany the police when executing the warrant in a home. Correct B The police need not always knock and announce their presence before entering a home. C Private citizens may execute the warrant. D Any items seized must be specified in the warrant.

When executing a warrant, the police do not need to always knock and announce their presence before entering a home. Usually, the police must knock and announce their authority and purpose and await admittance for a reasonable time or be refused admittance before using force to enter. However, no announcement needs to be made if the police reasonably suspect that knocking and announcing would be dangerous or futile or inhibit the investigation. Private citizens may NOT execute a warrant. Only the police may execute a warrant. A third party MAY be permitted to accompany the police when executing a warrant in a home. Typically the police may not be accompanied by the media or a third party. However, a third party may accompany the police if the third party is there to aid in executing the warrant, for example to identify stolen property that might be found in the home. It is not true that any items seized must be specified in the warrant. When executing a warrant, the police may seize any contraband or fruits or instrumentalities of a crime that they discover, regardless of whether they are specified in the warrant.

Which of the following statements regarding speedy trials is correct? response - correct Correct A One factor in determining whether a defendant's right to a speedy trial was violated is whether the defendant asserted his right. B Delays caused by counsel assigned by the court to the defendant should ordinarily be attributed to the state. C The remedy for a violation of the constitutional right to a speedy trial is dismissal without prejudice. D A defendant is entitled to speedy trial relief for the period between the dismissal of charges and later refiling.

Whether a defendant asserted his right is one factor in determining whether the defendant's right to a speedy trial was violated. The determination is made by an evaluation of the totality of the circumstances, and the following factors should be considered: (i) length of the delay, (ii) reason for the delay, (iii) whether the defendant asserted his right, and (iv) prejudice to the defendant. Delays caused by counsel assigned by the court to the defendant should ordinarily be attributed to the defendant and NOT to the state. The remedy for a violation of the constitutional right to a speedy trial is dismissal with prejudice. A defendant is NOT entitled to speedy trial relief for the period between the dismissal of charges and later refiling.

A municipal ordinance makes it illegal to text while driving. One day while on patrol in the municipality, a police officer pulled over a minivan full of teenagers because the driver was speeding. When the officer approached the minivan, he saw that the driver had his smartphone in his hand. The officer asked the driver: "Were you just texting, young man?" The driver replied, "No, I was just going to call my mom after you pulled me over to tell her I am going to be late now." The officer said: "Let me see your phone. And give me your driver's license and registration." The driver handed the officer his smartphone, along with his license and registration, and the officer started perusing the driver's text messages. Within no time, a nude photograph of one of the driver's passengers appeared in the texts. The driver was arrested, given Miranda warnings, and charged with possession of child pornography. At the police station, the police conducted an inventory search of the driver's belongings, and collected his smartphone, wallet, and car keys. At trial, the driver moved to suppress the evidence of the photograph found on his phone. How should the court rule on the motion? response - correct AGrant the motion because the officer needed to give the driver Miranda warnings before asking him whether he was texting. Correct BGrant the motion because the driver had a reasonable expectation of privacy in his text messages. CDeny the motion because the driver consented to the search of his smartphone. DDeny the motion because the smartphone was seized through an inventory search.

Without a warrant, the search of the driver's smartphone violated the driver's Fourth Amendment rights because a person has a reasonable expectation of privacy in his smartphone data. [Riley v. California, (2014)] Absent an exception, a warrantless search is considered per se unreasonable, and no exception to the warrant requirement applies to these facts. (A) is incorrect because officers may ask questions of the driver without issuing Miranda warnings. Miranda warnings are required only when a person is undergoing custodial interrogation. Generally, a person is in custody if he does not feel free to terminate the interrogation and leave and the interrogation is coercive in nature. Routine traffic stops are considered noncustodial in nature because they are brief and temporary, and, therefore, Miranda warnings are not required to ask questions of detained motorists. (C) is incorrect because the driver did not consent to the officer searching the data on his phone. The officer did not ask if he could see the driver's phone but rather demanded it: "Let me have your phone." By handing over his phone (which was requested along with his driver's license and registration), the teenager did not give the officer permission to look through his smartphone's data. (D) is incorrect because, even if the smartphone was legally obtained through an inventory search, the police would still need a warrant to view the texts and photographs on the phone.


संबंधित स्टडी सेट्स

39. Purine and Pyrimidine Metabolism

View Set

Lecture 3: Male reproductive tract-common species

View Set

chapter 23: Pharmacologic Pain Management- OB

View Set

InQuizitive: Chapter 11. Health and Society

View Set

Macro FINAL Ch. 11 practice quiz

View Set

FAMILY CHILD AND HUMAN DEVELOPMENT FINAL

View Set

2013 ICDC Business Administration Exam Quizlet

View Set